Clin Med Case Studies Exam

Réussis tes devoirs et examens dès maintenant avec Quizwiz!

A 62-year-old man with recently diagnosed emphysema presents to your office in November for a routine examination. He has not had any immunizations in more than 10 years. Which of the following immunizations would be most appropriate for this individual? A) Tetanus-diphtheria (Td) only B) Tdap, pneumococcal, and influenza C) Pneumococcal and influenza D) Tdap, pneumococcal, influenza, and meningococcal

B. In an adult with a chronic lung disease, one-time vaccination with pneumococcal vaccine and annual vaccination with influenza vaccine are recommended. A Tdap booster should be recommended to all adults who have not had a Td booster within 10 years and have never had a Tdap vaccine as an adult.

A 60-year-old man is diagnosed with moderately severe (stage II) COPD. He admits to a long history of cigarette smoking and is still currently smoking. In counseling him about the benefits of smoking cessation, which of the following statements is most accurate? A: By quitting, his pulmonary function will significantly improve. B: By quitting, his current pulmonary function will be unchanged, but the rate of pulmonary function decline will slow. C: By quitting, his current pulmonary function and the rate of decline are unchanged, but there are cardiovascular benefits. D: By quitting, his pulmonary function will approach that of a nonsmoker of the same age.

B. Smoking cessation will not result in reversal of the lung damage that has already occurred, but can result in a slowing in the rate of decline of pulmonary function. In fact, smoking cessation can result in the rate of decline returning to that of a nonsmoker.

A 56-year-old man is admitted to the hospital for chest pain of 2-hour duration. His heart rate is 42 bpm, with sinus bradycardia on ECG, as well as ST-segment elevation in leads II, III, and aVF. Which of the following is the most likely diagnosis? A) He is likely in good physical condition with increased vagal tone. B) He likely has suffered an inferior wall MI. C) He likely has an LV aneurysm. D) The low heart rate is a reflection of a good cardiac ejection fraction.

B. A 56 year old woman is admitted to the hospital with a 2 hour history of chest pain. Sinus bradycardia is often seen with inferior wall MI, because the right coronary artery supplies the inferior wall of the left ventricle and the sinoatrial node. The ischemic changes in leads II, III, and aVF are in the region of the inferior leads. Understanding which leads reflect which portion of the heart allows for an understanding of the aspect of the heart that is affected. Also understanding the area of the heart perfused by the various coronary arteries allows for correlation of associated symptoms or therapy.

A 37-year-old woman is noted to have gallstones on ultrasonography. She is placed on a low-fat diet. After 3 months she is noted to have severe right upper quadrant pain, fever to 102°F, and nausea. Which of the following is the most likely diagnosis? A) Acute cholangitis B) Acute cholecystitis C) Acute pancreatitis D) Acute perforation of the gallbladder

B. Acute cholecystitis is one of the most common complications of gallstones. This patient with fever, right upper quadrant pain, and a history of gallstones likely has acute cholecystitis. Acute cholangitis usually presents with Charcot's triad (RUQ pain, jaundice, and fever/chills) due to an ascending infection proximal to an obstructed bile duct.

Which of the following asymptomatic patients would most benefit from treatment of the finding of more than 105 CFU/mL of Escherichia coli on urine culture? A) A 23-year-old asymptomatic sexually active woman B) A 33-year-old asymptomatic pregnant woman C) A 53-year-old asymptomatic diabetic woman D) A 73-year-old asymptomatic woman in a nursing home

B. All of these patients are asymptomatic, and no benefit from treatment in terms of reduction in symptomatic UTIs or hospitalization has been shown for any of the other cases mentioned, except for pregnancy. Treatment is undertaken to prevent upper tract infection, preterm delivery, and possible fetal loss.

Which of the following are commonly seen in brain imaging of patients with Alzheimer disease? A) Chronic subdural hematoma B) Cortical atrophy with atrophy of medial temporal structures C) Ventriculomegaly without cortical atrophy D) Normal cerebral ventricles and normal brain tissue, acetylcholine deficiency

B. Alzheimer disease has no pathognomonic structural imaging criteria, but may include cortical atrophy and mesial temporal atrophy, whereas normal pressure hydrocephalus has enlarged brain ventricles without significant brain atrophy. Functional imaging can detect decreased perfusion and decreased metabolism in the temporal, parietal, and prefrontal cortex in patients with AD.

On routine blood work performed for a life insurance application, a 48-year-old premenopausal woman was found to have a calcium level of 12 mg/dL (normal = 8.8-10.4 mg/dL) and a phosphate level of 2 mg/dL (normal = 3.0-4.5 mg/dL). She is not anemic and has no symptoms. Her medical history is significant for osteoporosis, discovered on a dual-energy x-ray absorptiometry (DEXA) scan performed last year. Which of the following is the most likely cause of her hypercalcemia? A) Multiple myeloma B) Parathyroid adenoma C) Familial hypocalciuric hypercalcemia D) Sarcoidosis E) Undiagnosed breast cancer

B. An asymptomatic, most likely chronically elevated calcium level is most likely caused by primary hyperparathyroidism due to a parathyroid adenoma. The hypercalcemia is presumed to be chronic because she has osteoporosis and is premenopausal. Familial hypocalciuric hypercalcemia can also lead to elevated serum calcium and low serum phosphate levels, but is usually asymptomatic and is far more rare than primary hyperparathyroidism.

A 65-year-old man with a history of rheumatoid arthritis is found to have a microcytic anemia. He had a colonoscopy 1 year ago which was normal and stool guaiac is negative. Which of the following is the most likely cause of his anemia? A) Iron deficiency B) Chronic disease C) Pernicious anemia D) Folate deficiency

B. Anemia of chronic disease can cause normocytic or microcytic anemia, and may be secondary to rheumatoid arthritis in the patient. Iron-deficiency anemia is less likely with a normal colonoscopy and negative stool guaiac, and serum iron studies could be used to help differentiate the two.

A 21-year-old woman develops auditory hallucinations and persecutory delusions over the course of 3 days. She was hospitalized and started on haloperidol 2 mg three times daily. Within a week of treatment, she developed stooped posture and a shuffling gait. Her head was slightly tremulous and her movements became slowed. Her medication was changed to thioridazine, and trihexyphenidyl was added. Over the next 2 weeks, she became much more animated and reported no recurrence of her hallucinations. Which of the following is the most likely diagnosis? A) Hyperparathyroidism B) Adverse effect of neuroleptic C) Encephalitis D) Hypermagnesemia E) Tourette syndrome

B. Butyrophenones, the most commonly prescribed of which is haloperidol, routinely produce some signs of parkinsonism if they are used at high doses for more than a few days. This psychotic young woman proved to be less sensitive to the parkinsonian side effects of thioridazine than she was to haloperidol. Adding the anticholinergic drug trihexyphenidyl may have also helped to reduce the patient's symptoms.

A 25-year-old man with a history of a duodenal ulcer is noted to have a hemoglobin level of 10 g/dL. He does not report any visible GI blood loss. Which of the following most likely will be seen on laboratory investigation? A) Reticulocyte count of 4% B) Elevated total iron-binding capacity C) Normal serum ferritin D) Mean corpuscular volume of 105 fL

B. Chronic gastrointestinal blood loss leads to low ferritin levels reflecting diminished iron stores, elevated TIBC, and low iron saturation. There is a microcytic anemia (low MCV) with a low reticulocyte count. The reticulocyte count would be elevated with acute blood loss, but the patient has not experienced this.

A 55-year-old man with congestive heart failure develops bilateral pleural effusions. Which of the following is the most likely pleural fluid characteristic if thoracentesis is performed? A) Pleural fluid LDH 39, LDH ratio 0.2, protein ratio 0.7 B) Pleural fluid LDH 39, LDH ratio 0.2, protein ratio 0.1 C) Pleural fluid LDH 599, LDH ratio 0.9, protein ratio 0.1 D) Pleural fluid LDH 599, LDH ratio 0.9, protein ratio 0.7

B. Congestive heart failure is commonly associated with bilateral pleural effusions, which are transudative, as a consequence of alteration of Starling forces. The effusions of heart failure are best managed by treating the heart failure, for example, with diuretics, and typically do not require thoracentesis. Per Light Criteria, all other options would be classified as an exudative pleural effusion.

A 68-year-old man with a history of end-stage renal disease is admitted to the hospital for chest pain. On examination, a pericardial friction rub is noted. His ECG shows diffuse ST-segment elevation. Which of the following is the best definitive treatment? A) NSAIDs B) Dialysis C) Steroids D) Kayexalate (sodium polystyrene sulfonate)

B. Dialysis Uremic pericarditis is considered a medical emergency and an indication for urgent dialysis.

A 24-year-old intravenous drug user is admitted with 4 weeks of fever. He has three blood cultures positive with Candida spp and suddenly develops a cold blue toe. Which of the following is the appropriate next step? A) Repeat echocardiography to see if the large aortic vegetation previously seen has now embolized. B) Cardiovascular surgery consultation for aortic valve replacement. C) Aortic angiography to evaluate for a mycotic aneurysm, which may be embolizing. D) Switch from fluconazole to amphotericin B.

B. Fungal endocarditis, which occurs in intravenous drug users or immunosuppressed persons with indwelling catheters, frequently gives rise to large friable vegetations with a high risk of embolization (often to the lower extremities) and is very difficult to cure with antifungal medications. Valve replacement is usually necessary. Repeat echocardiography would not add to the patient's care because the clinical diagnosis of peripheral embolization is almost certain, and it would not change the management. Medical therapy with any antifungal agent is unlikely to cure this infection. Mycotic aneurysms may occur in any artery as a consequence of endocarditis and can cause late embolic complications, but in this case, the source probably is the heart.

A 56-year-old man has brittle diabetes (difficult to control with widely fluctuating blood sugars), tan skin, and a family history of cirrhosis. Select the cause (A-G) that is probably responsible for the patient's presentation. A) Wilson disease B) Hematochromatosis C) Primary biliary cirrhosis D) Sclerosing cholangitis E) Autoimmune hepatitis F) Alcohol-induced hepatitis G) Viral hepatitis

B. Hemochromatosis is a genetic disorder of iron metabolism. Progressive iron overload leads to organ destruction. Diabetes mellitus, cirrhosis of the liver, hypogonadotropic hypogonadism, arthropathy, and cardiomyopathy are among the more common end-stage developments. Skin deposition of iron leads to "bronzing" of the skin, which could be mistaken for a tan. Diagnosis is made early in the course of disease by demonstrating elevated iron stores but can be made through liver biopsy with iron stains. Genetic testing is available. Therapy involves phlebotomy to remove excess iron stores.

What would be the correct management of a person who presents with acute vision loss for 24 hours in one eye and suspected GCA? A) The vision is gone in the affected eye. There is nothing more to do. B) Start steroid medications immediately - either IV or oral - after confirming an elevated ESR and CRP to prevent vision loss in the contralateral eye. C) Arrange for a temporal artery biopsy in the next 24-48 hours. If positive, start steroids - either IV or oral. D) Draw labs to check for ESR, CRP, and platelets. Bring the patient back to clinic within one week and if results are positive, start steroids - either IV or oral.

B. In patients that have developed vision loss already in one eye and GCA is strongly suspected it is recommended to start steroids while the workup confirms or denies the suspicion of GCA. In patients presenting with no evidence of vision loss a temporal biopsy can be performed ASAP prior to starting medical treatment.

In the United States, which of the following is most likely to have caused the congestive heart failure in the patient described in Question 4.1 (55-year-old man is noted to have moderately severe congestive heart failure with impaired systolic function.)? A) Diabetes B) Atherosclerosis C) Alcohol D) Rheumatic heart disease

B. In the United States, the most common cause of CHF associated with impaired systolic function is ischemic cardiomyopathy due to coronary atherosclerosis.

A 48-year-old man presents to your clinic following a health fair screening of his cholesterol level because he was told that it is high. He has diet-controlled diabetes mellitus, plays tennis, exercises three to five times per week, and appears to be in good physical condition. He is a nonsmoker and has no family history of cardiovascular disease. His profile is total cholesterol 212 mg/dL, HDL 35 mg/dL, LDL 138 mg/dL, and triglycerides 175 mg/dL. Following a review of this patient's profile, which of the following would you recommend? A) Administer gemfibrozil. B) Administer HMG-CoA reductase inhibitor (statin). C) Administer low-dose niacin and slowly increase to achieve 3 g daily. D) Suggest he continues his current diet and exercise program.

B. In this scenario, although this 48-year-old man is very active with an exercise program, the key is that he has diabetes. A patient with diabetes is at high risk for development of atherosclerotic cardiovascular disease, and should be started on statins according to 2013 AHA/ACC guidelines.

A 22-year-old woman is pregnant and at 14-week gestation. Her hemoglobin level is 9 g/dL. She asks why she could have iron deficiency when she is no longer menstruating. Which of the following is the best explanation? A) Occult gastrointestinal blood loss B) Expanded blood volume and transport to the fetus C) Hemolysis D) Iron losses as a result of relative alkalosis of pregnancy

B. Iron deficiency occurs in pregnancy as a result of the expanded blood volume and active transport of iron to the fetus.

A 36-year-old woman has severe burning chest pain that radiates to her neck. The pain occurs particularly after meals, especially when she lies down, and is not precipitated by exertion. She is admitted for observation. Serial ECG and troponin I levels are normal. Which of the following is the best next step? A) Stress thallium treadmill test B) Initiation of a proton pump inhibitor C) Coronary angiography D) Initiation of an antidepressant such as a selective serotonin reuptake inhibitor E) Referral to a psychiatrist

B. It is appropriate to evaluate chest pain to first rule out cardiac ischemia. One of the most common causes of "chest pain" particularly in a younger patient is gastroesophageal reflux or esophageal spasm. This patient has classic symptoms of reflux esophagitis and is best treated with a proton pump inhibitor. If the chest pain has the characteristics of angina pectoris (substernal location, precipitated by exertion, relieved by rest or nitroglycerin), it should be investigated with a stress test or coronary angiography.

A 35-year-old man has undertaken a strict fad diet for 3 months. He previously had been healthy but now complains of fatigue. His hemoglobin level is 10 g/dL, and his MCV is 105 fL. Which of the following is the most likely etiology of his anemia? A) Iron deficiency B) Folate deficiency C) Vitamin B12 deficiency D) Thalassemia E) Sideroblastic anemia

B. Macrocytic anemia is usually a result of folate or vitamin B12 deficiency. Because vitamin B12 stores last for nearly 10 years, a dietary change of several months would more likely cause folate deficiency. Also vitamin B12 deficiency can lead to neurologic symptoms. Folate is found in green leafy vegetables. Iron deficiency, thalassemias, and sideroblastic anemias will likely be microcytic with a MCV <80.

A 63-year-old man is brought in by his family because of memory loss. They have noted a worsening of his symptoms over several months. They also report that he has had multiple falls, hitting his head on one occasion, and has had frequent urinary incontinence. On examination, a gait apraxia is noted. Which of the following is the most likely diagnosis? A) Alzheimer disease B) Normal pressure hydrocephalus C) Dementia with Lewy bodies D) Delirium

B. Normal pressure hydrocephalus classically causes dementia, incontinence, and gait disturbance. All of the other listed conditions may cause memory disturbance, but the constellation of these three symptoms is most consistent with normal pressure hydrocephalus.

A 52-year-old man complains of bilateral knee pain for about 1 year. He is noted to have a body mass index (BMI) of 40 kg/m2. Which of the following is the best therapy? A) Allopurinol B) Ibuprofen C) Methotrexate D) Intravenous ceftriaxone E) Oral glucocorticoids

B. Obesity is a risk factor for osteoarthritis, which is common in the knees and typically presents with a gradual onset and worsening of symptoms. Along with exercise and efforts to lose weight, an NSAID medication, such as ibuprofen, may provide symptomatic relief.

A 4-year-old boy presents with a 3-day history of fever with chills, cough, and fast breathing. His parents report decreased oral intake and increased difficulty breathing with retractions over the last 24 hours. His initial vital signs are heart rate 144 bpm; respiratory rate 32/min; temperature 101.3°F (38.5°C); oxyhemoglobin saturation 89% on room air. He is immediately started on supplemental oxygen, and his oxyhemoglobin saturation improves to 95%. Subsequent evaluation, including a chest X-ray, is suggestive of right middle and lower lobe pneumonia. What is the mechanism for the low oxyhemoglobin saturation in this patient? A. Hypoventilation B. Ventilation-perfusion mismatch C. Diffusion block D. Shunt E. Increased dead space in lungs

B. Of all the causes of hypoxemia in children, ventilation-perfusion mismatch is the most common in clinical situations. In a child with pneumonia, the middle and lower lobes of the right lung will continue to receive pulmonary blood flow, but the exchange of gases will be impaired due to poor air entry in those parts of the lung. This generates a mismatch between the ventilation (which is reduced) and perfusion (which is maintained), causing most of the blood from those areas to return to the left atrium in a relatively deoxygenated state. This leads to an overall drop in the percentage of hemoglobin molecules that are saturated with oxygen, manifesting as a drop in the measured oxyhemoglobin saturation. Hypoventilation occurs in patients with neuromuscular weakness and can cause significant elevation of carbon dioxide levels before it causes hypoxemia. Diffusion block is associated with lung disorders, such as idiopathic pulmonary fibrosis and interstitial lung diseases, which increase the thickness of the alveolar basement membrane, reducing the exchange of gases. A shunt causes hypoxemia due to the mixing of deoxygenated blood with oxygenated blood when the former has bypassed the lung completely (such as right-to-left shunt in a patient with Tetralogy of Fallot). A patient with pneumonia will not have increased dead space in the lungs.

A 62-year-old diabetic man underwent an abdominal aortic aneurysm repair 2 days ago. He is being treated with gentamicin for a urinary tract infection. His urine output has fallen to 300 mL over 24 hours, and his serum creatinine has risen from 1.1 mg/dL on admission to 1.9 mg/dL. Which of the following laboratory values would be most consistent with a prerenal etiology of his renal insufficiency? A) FENa of 3% B) Urinary sodium level of 10 mEq/L C) Central venous pressure reading of 10 mm Hg D) Gentamicin trough level of 4 μg/mL

B. Prerenal insufficiency connotes insufficient blood volume, typically with FENa less than 1% and urinary sodium less than 20 mEq/L. Supporting information would be a low central venous pressure reading (normal central venous pressure is 4-8 mm Hg). The gentamicin level of 4 μg/mL is elevated (normal <2 μg/mL) and may predispose to kidney damage.

A 63-year-old woman with a history of cervical cancer treated with hysterectomy and pelvic irradiation now presents with acute oliguric renal failure. On physical examination, she has normal jugular venous pressure, is normotensive without orthostasis, and has a benign abdominal examination. Her urinalysis shows a specific gravity of 1.010, with no cells or casts on microscopy. Urinary FENa is 2% and the Na level is 35 mEq/L. Which of the following is the best next step? A) Bolus of intravenous fluids B) Renal ultrasound C) Computed tomographic (CT) scan of the abdomen with intravenous contrast D) Administration of furosemide to increase her urine output

B. Renal ultrasound is the next appropriate step to assess for hydronephrosis and to evaluate for bilateral ureteral obstructions, which are common sites of metastases of cervical cancer. Her physical examination and urine studies (showing an FE >1%) are inconsistent with hypovolemia, so intravenous infusion is unlikely to improve her renal function. Use of loop diuretics may increase her urine output somewhat but does not help to diagnose the cause of her renal failure or to improve her outcome. Further imaging may be necessary after the ultrasound, but use of intravenous contrast at this point may actually worsen her

A 42-year-old overweight but otherwise healthy woman presents with sudden onset of right upper abdominal colicky pain 45 minutes after a meal of fried chicken. The pain is associated with nausea and vomiting, and any attempt to eat since then has caused increased pain. Which of the following is the most likely cause? A) Gastric ulcer B) Cholelithiasis C) Duodenal ulcer D) Acute hepatitis

B. Right upper abdominal pain of acute onset that occurs after ingestion of a fatty meal and is associated with nausea and vomiting is most suggestive of biliary colic as a result of gallstones. Duodenal ulcer pain is likely to be diminished with food, and gastric ulcer pain is not likely to have acute severe onset. Acute hepatitis is more likely to produce dull ache and tenderness.

MC Pneumonia? A) Salmonella spp B) S pneumoniae C) Parvovirus B19 D) Fat embolus E) Hematuria

B. S pneumoniae is the most common causative agent for pneumonia.

Which of the following is the most important first step in the treatment of diabetic ketoacidosis? A) Replacement of potassium B) Intravenous fluid replacement C) Replacement of phosphorus D) Antibiotic therapy

B. The basic tenets of treating DKA include intravenous fluid, insulin to control the glucose level, correction of metabolic disturbances (such as repletion of potassium), and identification of the underlying etiology.

A 69-year-old man complains of gradually worsening dyspnea and a nagging cough over the past 3 months but no fevers. He is found to have a right-sided pleural effusion, which is tapped and is grossly bloody. Which of the following is the most likely diagnosis? A) Parapneumonic effusion B) Malignancy in the pleural space C) Rupture of aortic dissection into the pleural space D) Pulmonary embolism with pulmonary infarction

B. The most common causes of hemorrhagic pleural effusion are trauma, malignancy, and pulmonary embolism. Pulmonary embolism would be suggested by acute onset of dyspnea and pleuritic chest pain rather than this subacute presentation. Similarly, aortic rupture can produce a hemothorax but would have an acute presentation with pain and hemodynamic compromise.

A 56-year-old man with known CKD presents with a 3-day history of shortness of breath and rapid weight gain. On examination, you are able to auscultate an S3, hear crackles at the bases, and see moderate jugular venous distension (JVD). Which of the following is your next step in evaluation? A) Perform an echocardiogram. B) Order a chest x-ray. C) Measure a Cr to calculate GFR. D) Check for cardiac enzymes.

B. The patient has CKD with volume overload as evidenced by symptoms and physical examination. A simple first step is to do a chest x-ray to confirm what you already suspect—pulmonary edema. After initiating furosemide (Lasix), the chest x-ray may be repeated to see to what degree the diuresis has improved the overload. Cardiac workup is also indicated but would not be the first test done.

A 67-year-old woman with known Parkinson disease is brought to the clinic by her health-care provider. She is confined to a wheelchair and completely dependent on others. You notice large grossly abnormal movements in both the arms and legs. The patient has to be strapped to the wheelchair to avoid falling out and can't keep her shoes on due to the jerking movements. Bed rails have had to be installed on her bed to prevent her from falling out at night. She is not able to tell the correct month or the year. She has not had a change in her medication in 6 months. Which of the following medication adjustments would benefit her most? A) Add haloperidol. B) Decrease levodopa/carbidopa. C) Increase levodopa/carbidopa. D) Add donepezil. E) Add entacapone.

B. The patient is suffering from dyskinesias from too much levodopa/carbidopa. Stopping levodopa/carbidopa is usually not an option for most patients; however, a reduction of the medication would be of the most benefit to her. Haloperidol would be a good choice if the patient was suffering from hallucinations. Donepezil is a medication used primarily for Alzheimer dementia and has no use in Lewy body dementia. Entacapone is a medication to enhance levodopa/carbidopa.

A 67-year-old man with coronary artery disease, dyslipidemia, and eczema comes to you complaining of lower back pain and left leg pain. The pain is worse when he stands for long periods of time, but improves when he bends forward to push his shopping cart around the grocery store. He indicates that his feet "burn" and "ache" after walking different distances every day. His lower extremity neuromuscular examination is unremarkable. Which of the following is the most appropriate treatment for this patient? A) Emergent spinal cord decompression B) Epidural corticosteroid injection C) Kyphoplasty D) Bed rest for 4 days E) Tramadol

B. The patient's history is classic for spinal stenosis. Often patients find relief by sitting or stooping. NSAIDs, physical therapy, and epidural corticosteroid injections are used to relieve pain. Surgical decompression is used in cauda equina syndrome, and kyphoplasty is useful in vertebral fractures. Bed rest is not used in the conservative treatment of back pain for any cause and has been shown to increase the duration of pain.

In a suicide attempt, an 18-year-old woman took 4 g of acetaminophen, approximately 8 hours previously. Her acetaminophen level is 30 μg/mL. Which of the following is the best next step to be performed for this patient? A) Immediately start N-acetylcysteine B) Observation C) Alkalinize the urine D) Administer intravenous activated charcoal

B. The serum acetaminophen level of 30 μg/mL, with last ingestion 8 hours previously, is plotted on the nomogram and falls below the "danger zone" of possible hepatic injury. Thus, this patient should be observed. Sometimes, patients will take more than one medication so that serum and/or urine drug testing may be worthwhile. Gastrointestinal activated charcoal, not intravenous charcoal, is used for other ingestions.

A 74-year-old man was noted to have excellent cognitive and motor skill 12 months ago. His wife noted that 6 months ago his function deteriorated noticeably, and 2 months ago another level of deterioration was noted. Which of the following is most likely to reveal the etiology of his functional decline? A) HIV antibody test B) Magnetic resonance imaging of the brain C) Cerebrospinal fluid (CSF) Venereal Disease Research Laboratory (VDRL) test D) Serum thyroid-stimulating hormone

B. The stepwise decline in function is typical for multi-infarct dementia, diagnosed by viewing multiple areas of the brain infarct.

A 48-year-old woman is noted to have atrial fibrillation with a ventricular heart rate of 140 bpm. She is feeling dizzy and dyspneic with a systolic blood pressure of 75/48 mm Hg. Which of the following is the most appropriate next step? A) Intravenous digoxin B) DC cardioversion C) Vagal maneuvers D) Intravenous diltiazem (Cardizem)

B. This individual has significant symptoms and hypotension caused by the atrial fibrillation and rapid ventricular rate; consequently, DC cardioversion is the treatment of choice.

A 20-year-old woman with heavy menses (MCV Ferritin TIBC RDW) A) Increased Decreased Increased Decreased B) Decreased Decreased Increased Increased C) Normal Increased Normal Normal D) Decreased Increased Normal Normal E) Increased Increased Decreased Increased

B. This laboratory finding is diagnostic of iron deficiency anemia (microcytic, low ferritin, high TIBC, high RDW).

A 62-year-old smoker with a known history of CHD presents for establishment of care. He has normal blood pressure. His LDL cholesterol is 105 mg/dL, HDL cholesterol is 28 mg/dL, and total cholesterol is 170 mg/dL. According to the NICE guidelines, what medication therapy should be initiated at this time? A) Ezetimibe B) Atorvastatin C) Niacin D) Gemfibrozil

B. This patient has known CHD and should be started on statin therapy.

A 55-year-old woman presents to your office for follow-up. She was discharged from the hospital 1 week ago following a heart attack. She has quit smoking since then and vows to stay off cigarettes forever. Her lipid levels are total cholesterol 240 mg/dL, HDL 50 mg/dL, LDL 150 mg/dL, and triglycerides 150 mg/dL. Which of the following is the most appropriate management at this time? A) Institute therapeutic lifestyle changes alone. B) Institute therapeutic lifestyle changes and start on a statin. C) Start on a statin. D) Institute therapeutic lifestyle changes and start on a statin and nicotinic acid.

B. This patient has known CHD, documented by her recent myocardial infarction. All guidelines recommend therapeutic lifestyle changes as an integral part of care. With known CHD, statin therapy is also indicated. ACC/AHA guidelines would recommend high-intensity statin therapy.

A 50-year-old man has been treated for rheumatoid arthritis for many years. He currently is taking corticosteroids for the disease. On examination, he has stigmata of rheumatoid arthritis and some fullness on his left upper abdomen. His platelet count is slightly low at 105,000/mm3. His white blood cell count is 3100/mm3 with neutropenia, and hemoglobin level is 9 g/dL. Which of the following is the most likely etiology of the thrombocytopenia? A) Steroid induced B) Splenic sequestration C) Autoimmune destruction D) Prior gold therapy

B. This patient is likely suffering from Felty's syndrome characterized by rheumatoid arthritis, neutropenia, and splenomegaly. Splenomegaly from any etiology may cause sequestration of platelets, leading to thrombocytopenia.

A 62-year-old woman presents for a routine physical examination. She is asymptomatic and is not taking any medications. Her blood pressure is found to be 145/85 mm Hg on two readings and her body mass index (BMI) is 29. Review of her chart reveals that her blood pressure was 143/84 mm Hg on a visit 4 months ago for a urinary tract infection. Which of the following is the most accurate statement regarding her blood pressure? A) Her blood pressure is normal and she is at average risk for developing hypertension. B) She is at risk for needing pharmacologic treatment for hypertension. C) She has hypertension and should be started on a thiazide diuretic. D) She has hypertension and should be started on multidrug therapy.

B. This patient's blood pressure falls within the definition of hypertension but outside the need for immediate pharmacologic intervention. She would benefit from the institution of lifestyle modifications to try to reduce her risk of progression.

A 55-year-old man with history of hypertension and diabetes presents with intermittent nighttime cough for a few months. He states he often has a "weird taste" in his mouth a couple of hours after eating and is afraid of eating dinner because he gets terrible heartburn during the night. He states he has tried over the counter antacid and this has worked to somewhat alleviate his symptoms; however, his nighttime cough is still very bothersome. His vitals in office are within normal limits and physical examination is positive for epigastric tenderness upon palpation. Which one of the following is true regarding the most likely etiology of this patient's cough? A) It is the second leading cause of chronic cough. B) The most sensitive and specific test for this condition is a 24-hour esophageal pH monitoring. C) The first line of treatment for this condition is a trial of 4 weeks of H2 blocker. D) This condition always requires a diagnostic test for confirmation and should not be diagnosed clinically.

B. This patient's cough is most likely secondary to GERD. The most definitive test to diagnose this condition is a 24-hour pH monitoring test, however, this is not required for diagnosis. GERD is almost always a clinical diagnosis and a 4-week trial of proton pump inhibitor is both diagnostic and therapeutic. Lastly, GERD is the third leading cause of chronic cough, after upper airway cough syndrome and asthma.

A young woman presents to your office complaining of dizziness. When asked to describe the feeling, she gives a vague story of just feeling like "her head is too big" and she "feels like is not really here." The feeling is associated with palpitations, sweating, and nervousness and typically resolves in less than an hour. Her examination, including neurologic evaluation, is completely normal. Which of the following is the best next step? A) Magnetic resonance imaging (MRI) brain scan. B) Obtaining a thorough psychosocial history. C) Dix-Hallpike maneuver. D) Prescribe meclizine. E) Referral to neurology department.

B. This young woman is not describing vertigo. The word "dizzy" can mean several different things, so it is extremely important when obtaining the history to have the patient describe, as best he or she can, what is meant by "dizzy." Patients with vertigo often use descriptors indicating movement, such as "the room is moving around me" or "I'm on a roller coaster." Vague symptoms, or "out-of-body" experiences, or feeling disconnected from the environment, such as this young woman describes, are not typical of vertigo and indicate another problem. It would be important to know what the symptoms are associated with; for example, is there increased stress in her job or intimate relationship? Is this panic disorder or anxiety disorder?

A 5-year-old child is brought into the office due to the mother's concern of difficulty breathing. On examination, the child appears toxic, has a high fever, cough productive of thick mucopurulent expectorant, and stridor with wheezing. Which of the following is the most likely condition and organism that requires antibiotic therapy? A) Epiglottis-H influenzae B) Tracheitis—S aureus C) Epiglottis—S pyogenes (β-hemolytic Streptococcus group A) D) Tracheitis—S pyogenes (β-hemolytic Streptococcus group A) E) Retropharyngeal abscess—S aureus

B. Tracheitis matches the symptom description and is usually caused by S aureus. Group A streptococci are now the leading cause of epiglottis, but the symptom constellation is more likely due to tracheitis. Retropharyngeal abscess usually presents insidiously with neck pain with swelling, fever, dysphagia, and drooling.

A 59-year-old man with a known history of COPD presents with worsening dyspnea. On examination, he is afebrile. His breath sounds are decreased bilaterally. He is noted to have jugular venous distension (JVD) and 2+ pitting edema of the lower extremities. Which of the following is the most likely cause of his increasing dyspnea? A: COPD exacerbation B: Pneumonia C: Cor pulmonale D> Pneumothorax

C. JVD and lower extremity edema are suggestive of cor pulmonale, which is right heart failure due to chronically elevated pressures in the pulmonary circulation. Right heart failure causes increased right atrial pressures and right ventricular end-diastolic pressures, which then lead to liver congestion, jugular venous distension, and lower extremity edema.

A 49-year-old sedentary man has made an appointment because his best friend died of an MI at age 50. He asks about an exercise and weight loss program. In counseling him, which of the following statements regarding exercise is most accurate? A) To be beneficial, exercise must be performed every day. B) Walking for exercise has not been shown to improve meaningful clinical outcomes. C) Counseling patients to exercise has not been shown consistently to increase the number of patients who exercise. D) Intense exercise offers no health benefit over mild to moderate amounts of exercise.

C. The benefits of exercise are clear. Exercise decreases cardiovascular risk factors, increases insulin sensitivity, decreases the incidence of the metabolic syndrome, and decreases cardiovascular mortality regardless of obesity. The benefits of counseling patients regarding exercise are not so clear and counseling does not seem to increase the number of patients who exercise.

A 38-year-old woman presents with progressively worsening dyspnea and cough. She has never smoked cigarettes, has no known passive smoke exposure, and does not have any occupational exposure to chemicals. Pulmonary function testing shows obstructive lung disease that does not respond to bronchodilators. Which of the following is the most likely etiology? A: Radon exposure at home B: COPD C: α1-Antitrypsin deficiency D: Asthma

C. This patient has a fixed airway obstruction consistent with COPD. The airway obstruction of asthma would be at least partially reversible on testing with a bronchodilator. α1-Antitrypsin deficiency should be considered in a patient who develops COPD at a young age, especially if there is no other identifiable risk factor.

A 68-year-old patient of your practice with known COPD has pulmonary function testing showing an FEV1 of 40% predicted has been having frequent exacerbations of his COPD. His SaO2 by pulse oximetry is 91%. Which of the following medication regimens is the most appropriate? A: Inhaled salmeterol BID and albuterol as needed B: Oral albuterol daily and inhaled fluticasone BID C: Inhaled fluticasone BID, inhaled tiotropium BID, and inhaled albuterol as needed D: Inhaled fluticasone BID, inhaled tiotropium BID, inhaled albuterol as needed, and home oxygen therapy

C. This patient has stage III COPD with frequent exacerbations. He is best treated by a long-acting bronchodilator (eg, tiotropium) and an inhaled steroid (eg, fluticasone) used regularly, along with an inhaled, short-acting bronchodilator on an as-needed basis.

A 65-year-old woman with a history of benign positional vertigo returns to your office for follow-up. Although manageable, the symptoms of vertigo continue to recur periodically. Between episodes she generally feels normal but occasionally somewhat "off-balance." Today, her neurologic examination is completely normal, except that the thresholds of both air and bone conduction of a vibrating 256-Hz tuning fork are elevated on the left side. Which of the following is the most likely diagnosis? A) Intermittent benign positional vertigo B) Otosclerosis C) Acoustic neuroma D) Acute basilar artery infarct E) Panic disorder

C. Acoustic neuromas are slow-growing tumors of the eighth cranial nerve. Because of the slow growth of the tumor, the neurologic system often is able to accommodate, so patients may have only subtle symptoms that at first may be confused with benign positional vertigo. The keys in this patient's history are the persistent low-grade feelings of dysequilibrium and the finding of probable sensorineural hearing loss on the left side. This finding indicates a possible problem with the eighth nerve, and an MRI would best delineate the anatomy.

Which substance is likely to cause the most severe chemical burn in the eye? A) Nail polish B) Battery acid C) Drano (lye) D) Pepper spray E) Vinegar

C. Although acid can cause a severe surface burn, base substances appear to penetrate deeper in the eye structures. All exposures however, would need immediate irrigation.

A 36-year-old man is noted to have a bothersome "mole" that on biopsy reveals malignant melanoma. The pathologist comments that this histology is a very rare type of melanoma and usually escapes diagnosis until a more advanced stage. Which of the following is the most likely finding? A) Melanoma in situ B) Superficial spreading melanoma C) Amelanotic melanoma D) Nodular melanoma E) Acral lentiginous melanoma

C. Amelanotic melanoma is an uncommon type of melanoma that due to its lack of pigmentation often goes undiagnosed until it is in a more invasive and advanced stage.

Which of the following is the most likely explanation for multiple drug resistance to antibiotics that spreads from one type of bacteria to another? A) Adaptation B) Decreased bioavailability C) Gene transfer D) Mutation

C. Antibiotic drug resistance can occur through bacterial cell mutation, adaptation, or gene transfer. The best route for multiple drug resistance that spreads from one type of bacteria to another is via plasmid or chromosomal gene transfer.

A 21-year-old woman presents for her first Pap smear. She received the full HPV vaccine series at age 19. Assuming that her examination and Pap smear results are normal, when would you recommend that she return for a follow-up Pap smear? A) 6 months, as the first Pap smear should be followed up soon to reduce the false-negative rate associated with this screening test B) 1 year, as she is higher risk because of her age C) 3 years, as the Pap smear was normal D) 5 years, as she is at low risk because she received the HPV vaccine

C. As per USPSTF, screening for cervical cancer should begin at age 21 and be repeated at 3-year intervals. Six months and 1-year intervals are inappropriate and not part of routine screening recommendations for women with normal Pap smears. The use of HPV vaccine is not an indication to alter cervical cancer screening recommendations at this time.

A 59-year-old male smoker complains of severe substernal squeezing chest pain of 30-minute duration. The paramedics have given sublingual nitroglycerin and oxygen by nasal cannula. His blood pressure is 110/70 mm Hg and heart rate is 90 bpm on arrival to the ER. The ECG is normal. Which of the following is the best next step? A) Echocardiography B) Thallium stress test C) Aspirin D) Coronary angiography E) Coronary artery bypass

C. Aspirin is the first agent that should be used after oxygen and nitroglycerin. Aspirin use decreases mortality in the face of an acute coronary event. Because initial ECGs and cardiac enzymes may be normal in an acute MI, serial studies are needed to definitively rule out MI. Clinical assessment to exclude other causes of chest pain should be undertaken. The other answer choices are aimed toward diagnostic tests and may be important, but the first and foremost priority should be to "save myocardium."

A 78-year-old woman is diagnosed with early Alzheimer disease. Which of the following agents is most likely to help with the cognitive function? A) Haloperidol B) Estrogen replacement therapy C) Donepezil D) High-dose vitamin B12 injections

C. Cholinesterase inhibitors help with the cognitive function in Alzheimer disease and may slow the progression. Cholinesterase inhibitors are considered first line therapy.

A 50-year-old man with severe rheumatoid arthritis (MCV Ferritin TIBC RDW) A) Increased Decreased Increased Decreased B) Decreased Decreased Increased Increased C) Normal Increased Normal Normal D) Decreased Increased Normal Normal E) Increased Increased Decreased Increased

C. Chronic disease generally leads to a normocytic anemia with elevated ferritin level (acute-phase reactant); although a microcytic anemia can also be seen, a normocytic anemia is more common.

A 68-year-old man is hospitalized with Streptococcus bovis endocarditis of the mitral valve and recovers completely with appropriate therapy. Which of the following is the most important next step? A) Good dental hygiene and proper denture fitting to prevent reinfection of damaged heart valves from oral flora. B) Repeat echocardiography in 6 weeks to ensure the vegetations have resolved. C) Colonoscopy to look for mucosal lesions. D) Mitral valve replacement to prevent systemic emboli such as cerebral infarction.

C. Colonoscopy is necessary because a significant number of patients with S bovis endocarditis have a colonic cancer or premalignant polyp, which leads to seeding of the valve by gastrointestinal (GI) flora. Heart valves damaged by endocarditis are more susceptible to infection, so good dental hygiene is important, but in this case, the organism came from the intestinal tract, not the mouth, and the possibility of malignancy is most important to address. Serial echocardiography would not add to the patient's care after successful therapy because vegetations become organized and persist for months or years without late embolization. Prophylactic valve replacement would not be indicated because the prosthetic valve is even more susceptible to reinfection than the damaged native valve and would actually increase the risk of cerebral infarction or other systemic emboli as a consequence of thrombus formation, even if adequately anticoagulated.

Which of the following statements regarding CML is true? A) Peripheral smear shows elevated WBC count with mature and immature granulocytes, toxic granulation, and high LAP score. B) Usually presents initially with splenomegaly, anemia, and thrombocytopenia. C) Chromosomal translocations, most often t(9;22), are found in 90% to 95% of patients. D) Is an indolent disease, and should be monitored without treatment until patients enter accelerated or blast phase.

C. Definitive diagnosis of CML is established by demonstrating the presence of the Philadelphia chromosome or the underlying t(9;22) translocation, the BCR-ABL1 fusion gene or mRNA fusion product, which is found in nearly all patients. Toxic granulation and high LAP score are features of leukemoid reaction. Splenomegaly is common in CML, but significant cytopenias are not seen. Before imatinib and other TKIs, median survival in CML was 4 years with progression to blast (acute leukemic) phase and death. Imatinib or other TKIs are indicated as initial treatment for patients in chronic phase, with the goals of achieving remission and preventing progression of disease.

A 40-year-old man travels to Mexico and develops diarrhea 1 day after coming back to the United States. Which of the following is the most likely etiology of the symptoms? A) Rotavirus B) Giardia C) E coli D) S aureus E) Cryptosporidium

C. E coli is the most common etiology for traveler's diarrhea.

A 22-year-old woman at 24 weeks gestation presents with a 3-day history of a nonproductive cough and fever. She states she has been battling an upper respiratory infection that does not seem to go away. Initially, she thought that she had a cold, but the symptoms persisted. She was told by her obstetrician that she has the flu and to drink plenty of fluids and get some rest. Vital signs reveal temperature 101.2°F, blood pressure 120/80 mm Hg, heart rate 110/bpm, and respiratory rate 22 breaths/min. On physical examination, she appears ill; lungs exhibit wheezing. What would be an appropriate treatment for this patient? A. Doxycycline B. Ciprofloxacin C. Erythromycin D. Amoxicillin E. Penicillin

C. ERYTHROMYCIN. The clinical picture is suggestive of an atypical community-acquired pneumonia. Of the antibiotics listed, those that cover M. pneumoniae are doxycycline and erythromycin. Community-acquired pneumonia is defined as pneumonia diagnosed outside of the hospital. Doxycycline is an antibiotic used to treat a number of bacterial and protozoan infections. It is used in the treatment of bacterial pneumonia but is contraindicated in pregnancy. Ciprofloxacin is an antibiotic used in the treatment of bacterial infections but is contraindicated in pregnancy. Amoxicillin and penicillin are not effective against M. pneumoniae infections.

What is the best diagnostic test for a patient with suspected Cushing syndrome (ACTH-producing adenoma)? A) Random cortisol level B) ACTH-stimulation test C) Overnight 1-mg dexamethasone suppression test D) Pituitary MRI

C. Elevated cortisol greater than 5 μg/dL in the morning after a dose of dexamethasone at night indicates autonomous ACTH production (failure to be suppressed with dexamethasone). ACTH stimulation test is for adrenal insufficiency. Cortisol levels vary throughout the day, and are only useful when elevated to exclude adrenal insufficiency. Most ACTH-producing pituitary tumors are less than 5 mm and may not be seen on MRI.

At a routine checkup, a 6-year-old boy is found to have a blood pressure of 150/90 mm Hg. Repeated blood pressure readings are consistently elevated. The child was delivered at 36 weeks by normal spontaneous vaginal delivery with no complications. All major milestones were met on time and he currently is enrolled in first grade. The child has been healthy up until this point. Which of the following is the most appropriate diagnosis and therapeutic step? A) The child has essential hypertension and should be started on the DASH diet. B) The child most likely has hyperthyroidism and should be started on a β-blocker while thyroid studies are performed. C) The child most likely has renal parenchymal disease and should have a urinalysis and renal ultrasound ordered. D) The child most likely has "white coat" hypertension and the readings should be ignored if there is no family history of hypertension. E) The child most likely has a pheochromocytoma and should start a 24-hour urine collection for metanephrines.

C. Essential hypertension is rarely found in children less than 10 years of age and should be a diagnosis of exclusion. The most common cause of hypertension is renal parenchymal disease, and a urinalysis, urine culture, and renal ultrasonography should be ordered for all children presenting with hypertension.

A 39-year-old man is noted to have a deep venous thrombosis without any known risk factors. He notes that his brother also developed a pulmonary embolism at age 45, and his mother developed a "clot in the leg" when she was in her thirties. Which of the following is the most likely inherited disorder in this patient? A) Protein S deficiency B) Antithrombin III deficiency C) Factor V Leiden mutation D) Antiphospholipid antibody syndrome E) Familial malignancy syndrome

C. Factor V Leiden mutation is the most common hereditary thrombophilia. It is inherited in an autosomal dominant fashion and therefore will affect both men and women.

A 35-year-old man with hypertension presents with the sudden onset of right big toe pain. Which of the following is the best treatment? A) Ibuprofen B) Methotrexate C) Colchicine D) Intravenous antibiotics

C. Gouty arthritis often initially presents in the big toe ("podagra") and the use of HCTZ, a common treatment for hypertension, also can increase the risk. Colchicine can provide effective acute treatment.

A 44-year-old woman is noted to be nervous and has heat intolerance. Her thyroid gland is diffusely enlarged, nontender, with an audible bruit. Her TSH level is very low. Which of the following is the most likely etiology? A) Lymphocytic thyroiditis B) Hashimoto thyroiditis C) Graves disease D) Multinodular toxic goiter

C. Graves disease is the most common cause of hyperthyroidism in the United States. It often includes the thyroid gland features described, as well as the distinctive eye findings.

A 26-year-old man presents with fever, dysuria, and left knee pain. He reports being sexually active with a new partner as recently as 2 weeks ago. On physical examination, he is febrile and his left knee is erythematous, swollen, and tender. He denies a previous history of arthritis. Which of the following is the next best step? A) CBC with differential B) X-ray of the knee C) Aspiration of synovial fluid D) Serum uric acid level

C. Infectious arthritis would need to be high on the differential diagnosis because of the danger of gonococcal arthritis. The history supports this diagnosis. This patient needs a joint aspiration to look for gram-negative diplococci, crystals, and to obtain a sample for culture. He will likely require surgical drainage of the swollen joint and IV antibiotic therapy.

When initially evaluating a person with exposure of a chemical substance to the eye, what should be done first? A) Get a good history to help determine what kind of substance got in the eye B) Check the vision, pressure, and do a slit lamp exam to determine the extent of damage done by the substance C) Begin immediate ocular irrigation D) Getting the patient some oral pain medications so the patient is more comfortable and able to undergo an ocular evaluation

C. It is key to start irrigation of the ocular surface as soon as possible in patients with chemical exposure to the eye.

A 65-year-old cigarette smoker with a history of hypertension and mild congestive heart failure presents to the emergency room with worsening cough, fever, and dyspnea at rest. The illness began 1 week ago with fever, muscle aches, abdominal pain, and diarrhea, with nonproductive cough developing later that week and rapidly becoming worse. Therapy for which of the following atypical organisms must be considered in this case? A) C pneumoniae B) M pneumoniae C) Legionella pneumophila D) Coccidioidomycosis E) Aspergillus fumigatus

C. Legionella typically presents with myalgias, abdominal pain, diarrhea, and severe pneumonia. This patient is also more susceptible to Legionella given his smoking status. C. pneumoniae would be more likely in an older patient with a more gradual onset of symptoms and associated pharyngitis, hoarseness and/or sinus involvement. M. pneumonia are more common in a young adults and a common tested association is with bullous myringitis (blisters seen on the tympanic membrane). Coccidioidomycosis is endemic to the southwest United States and often causes a subclinical infection often after dust exposure. Aspergillus is more common in immunocompromised individuals and can present with hemoptysis and lung infarction.

A 73-year-old woman presents for concern about several tan-colored moles on her arms, face, and ears that have progressively grown over the past 6 months. Upon further examination, the moles are determined to be between 6 and 8 mm with very irregular borders. The physician decides to obtain an excisional biopsy. Which of the following skin lesions should the physician be most suspicious of based on history alone? A) Benign nevus B) Superficial spreading melanoma C) Lentigo maligna D) Nodular melanoma E) Acral lentiginous melanoma

C. Lentigo maligna is most often found in the elderly usually on chronic sun-damaged skin such as the face, ears, arms, and upper trunk. Think of this type with tan-colored lesions on sun-damaged skin that has very irregular borders.

A patient with known asthma undergoing therapy with inhaled corticosteroids and intermittent (short-acting) β2-agonist presents with complaints of nocturnal awakenings secondary to cough and occasional wheezing. This episode occurs three to four times per week. Pulmonary function tests in the past have shown mild obstructive lung disease. Which of the following is the best next step? A) Oral steroids B) Leukotriene inhibitors C) Long-acting β2-agonists D) Theophylline E) Antireflux therapy

C. Long-acting β2-agonists are indicated in this situation. The asthma would be classified as moderate persistent, and the recommended treatment is addition of long-acting β2-agonists (such as salmeterol) to the inhaled corticosteroids, which are particularly helpful with nocturnal symptoms.

A 45-year-old man presents with 3 days of watery diarrhea and abdominal cramping. He has no sick contacts and has not traveled recently. He is not currently taking any medications, but he was prescribed amoxicillin 2 weeks ago for a sinus infection. Which of the following is the treatment of choice for his diarrhea? A) Ciprofloxacin B) Azithromycin C) Metronidazole D) Loperamide

C. Metronidazole or oral vancomycin can be used to treat C difficile. Ciprofloxacin and azithromycin can be used for treatment of traveler's diarrhea. Loperamide can decrease the frequency of bowel movements but is contraindicated in any patient with suspected C difficile colitis.

Which of the following statements regarding breast cancer screening is true? A) Breast self-examination (BSE) has been shown to decrease mortality rates from breast cancer. B) Clinical breast examination (CBE) in conjunction with routine mammography is shown to improve mortality rates. C) Most abnormalities found on routine mammography are not breast cancer. D) Because breast cancer rates increase in older women, there is no upper age at which breast cancer screening may be discontinued.

C. Most abnormalities seen on mammography are not cancerous. They may, however, require further imaging studies, testing, or biopsy. Breast self-examination has not been definitively shown to reduce cancer mortality. Clinician breast examination may be of benefit but likely does not impact outcome if mammography is available. The age to consider discontinuation of mammography screening should be individualized, based on the woman's risk factors and overall health status.

A 39-year-old woman with multiple medical problems has been noted to have progressively worsening renal insufficiency. Which of the following measures is most important in the prevention of end-stage renal disease? A) Tobacco cessation B) Triglyceride control C) Glycemic control D) Weight control E) Dietary sodium restriction

C. Optimal control of high blood pressure, acidosis, volume depletion, and cholesterol are all important to prevent worsening renal function. Diabetes is a leading cause of end-stage renal disease. Tight glycemic control can prevent the microvascular complications of diabetes such as diabetic nephropathy, though it has not been shown to decrease significantly the occurrence of macrovascular complications of diabetes such as coronary artery disease (CAD) or peripheral vascular disease (PVD). Treating secondary hyperparathyroidism prevents complications such as renal osteodystrophy. The patient's weight does not impact on renal function substantially. Smoking has numerous health risks but does not tend to impact kidney function directly; nevertheless, its effect on cardiovascular system may impact on the kidneys.

Aplastic crisis A) Salmonella spp B) S pneumoniae C) Parvovirus B19 D) Fat embolus E) Hematuria

C. Parvovirus B19 is associated with aplastic crisis, especially in individuals with sickle cell disease.

Which of the following patients is the best candidate for lifestyle modification alone rather than lipid-lowering medications? A) A 60-year-old diabetic male smoker with a recent myocardial infarction: cholesterol 201 mg/dL, HDL 47 mg/dL, and LDL 138 mg/dL B) A 62-year-old diabetic man: cholesterol 210 mg/dL, HDL 27 mg/dL, and LDL 146 mg/dL C) A 57-year-old asymptomatic woman: cholesterol 235 mg/dL, HDL 92 mg/dL, and LDL 103 mg/dL D) A 39-year-old man with nephrotic syndrome: cholesterol 285 mg/dL, HDL 48 mg/dL, LDL 195 mg/dL

C. Patient A is at highest risk for future events because he has established CHD and diabetes, he smokes, and he recently had a myocardial infarction. Patient B has diabetes, a CHD equivalent. Besides lifestyle modifications, he should start drug therapy to lower his LDL and raise his HDL. Patient C has very high HDL, which is protective, and probably contributes to her elevated total cholesterol. Patient D has nephrotic syndrome causing hyperlipidemia, which may be treated by reduction of proteinuria using angiotensin-converting enzyme (ACE) inhibitors but often requires drug therapy such as statins.

A 30-year-old woman with ITP has been taking maximum corticosteroid doses and still has a platelet count of 20,000/mm3 and frequent bleeding episodes. Which of the following should she receive before her splenectomy? A) Washed leukocyte transfusion B) Intravenous interferon therapy C) Pneumococcal vaccine D) Bone marrow radiotherapy

C. Patients who undergo splenectomy are at risk for infections of encapsulated organisms such as Streptococcus pneumoniae and thus should receive the pneumococcal vaccine. It usually is given at least 2 weeks prior to splenectomy so that the spleen can help in forming a better immune response.

A 68-year-old white female presents to the ED with palpitations that started several hours ago. On physical examination she has an irregular heart beat with a rate of 75 to 90 bpm and a thready pulse. ECG shows atrial fibrillation. Her past medical history is significant for Type 2 diabetes diagnosed 10 years ago and a cholecystectomy last year. What therapy improves survival in patients with chronic atrial fibrillation? A) Pharmacologic cardioversion B) Rate control with nodal blocking agents C) Rhythm control with amiodarone D) Electrical cardiversion

C. Prevention of stroke is the only treatment that improves survival in atrial fibrillation in hemodynamically stable patients. Rate control (B) is preferred to rhythm control (C), but only because it is associated with fewer medication-related side effects and fewer hospitalizations. Cardioversion (A) and (D) does not offer survival benefits any greater than rate control.

A 56-year-old woman who presented with complaints of pruritus and fatigue has elevated alkaline phosphatase. Select the cause that is probably responsible for the patient's presentation. A) Wilson disease B) Hematochromatosis C) Primary biliary cirrhosis D) Sclerosing cholangitis E) Autoimmune hepatitis F) Alcohol-induced hepatitis G) Viral hepatitis

C. Primary biliary cirrhosis is thought to be an autoimmune disease leading to destruction of small- to medium-size bile ducts. Most patients are women between the ages of 35 and 60, who usually present with symptoms of pruritus and fatigue. Alkaline phosphatase elevated two to five times above the baseline should raise suspicion; diagnosis is confirmed with antimitochondrial Ab.

A 58-year-old woman is noted to have Graves disease and has a small goiter. Which of the following is the best therapy? A) Long-term propranolol B) Lifelong oral propylthiouracil (PTU) C) Radioactive iodine ablation D) Surgical thyroidectomy

C. Radioactive iodine is a definitive treatment for Graves disease. Surgery is indicated for obstructive symptoms, or for women during pregnancy. Propranolol is a good initial option to control tachycardia but not a long term option. PTU is a second line option due to the risk of hepatocellular necrosis.

A 25-year-old woman complains of pain in her PIP and metacarpophalangeal (MCP) joints and reports a recent positive antinuclear antibody (ANA) laboratory test. Which of the following clinical features would NOT be consistent with a diagnosis of SLE? A) Pleural effusion B) Malar rash C) Sclerodactyly D) Urinary sediment with RBC casts

C. Sclerodactyly, which is thickened and tight skin of the fingers and toes, is a classic feature of patients with scleroderma (who may also have a positive ANA test), but is not seen in SLE. The other findings (malar rash, serositis, glomerulonephritis) are typical of SLE, but not seen in scleroderma.

A 35-year-old woman complains of calf tenderness and acute dyspnea. The arterial blood gas reveals partial pressure of oxygen (PO2) of 76 mm Hg. Which of the following is the most common physical examination finding of pulmonary embolism? A) Wheezing B) Increased pulmonary component of the second heart sound C) Tachypnea D) Calf swelling E) Pulmonary rales

C. Tachypnea is the most common physical sign associated with pulmonary embolus. Calf or thigh pain and/or swelling occurs less frequently than tachypnea. Other common clinical manifestations of pulmonary embolus in decreasing frequency include pleuritic pain, cough, and orthopnea.

Based on the Ottawa Ankle Rules, which of the following examination findings would make obtaining a radiograph most appropriate? A) An 18-year-old athlete who injures his ankle during basketball but continues to play until the end of the game B) A 33-year-old overweight woman who has multiple injuries, including ankle pain, following a motor vehicle accident C) A 43-year-old man injured his ankle yesterday while playing volleyball and was unable to walk on it immediately afterwards D) A 22-year-old woman seen in the emergency room (ER) immediately after falling while drunk and who falls asleep repeatedly during the examination

C. The Ottawa Ankle Rules state that radiography should be obtained if there is pain in the malleolar area with point tenderness over the tip of lateral or medial malleolus, bony tenderness over area of base of fifth metatarsal or navicular bone, or an inability to bear weight for four steps immediately after the injury (answer choice C). These decision rules are not validated in patients with multiple painful injuries or with an altered mental status.

A 52-year-old woman presents to the office for an acute visit complaining of 2 hours of headache. She says that it came on suddenly with no account of trauma and is the worst headache she has ever had. She has had migraines since she was an early adult. The pain is described as "stabbing" and is more severe on the left side. She takes no medications and recently stopped taking oral contraceptive pills after going through menopause. Her blood pressure is elevated at 145/95 mm Hg, but otherwise she has no focal neurologic abnormalities on examination. She is alert and oriented to person, place, time, and situation. Which of the following is the most appropriate management at this time? A) Prescribe a triptan medication. B) Schedule a noncontrast head CT scan for tomorrow morning. C) Call 911 and transfer the patient to the nearest emergency room. D) Prescribe an antihypertensive medication and follow up in 2 weeks.

C. The acute onset of the most severe headache in a patient's life is concerning for the presence of a subarachnoid hemorrhage. This is a medical emergency. This patient should be transported by emergency medical services to the nearest emergency facility for stabilization and management.

Which of the following laboratory findings is most consistent with poststreptococcal glomerulonephritis? A) Elevated serum complement levels B) Positive antinuclear antibody titers C) Elevated ASO titers D) Positive blood cultures E) Positive cryoglobulin titers

C. The antistreptolysin-O titers typically are elevated and serum complement levels are decreased in poststreptococcal GN. Answer B would be more likely to be seen in a patient with Lupus Nephritis along with decreased complement levels (C3 and C4). Answer D is more likely in a patient with glomerulonephritis secondary to endocarditis where valvular disease would also be present. Answer E is appropriate for glomerulonephritis secondary to cryoglobulinemia where the patient is also likely to test positive for Hepatitis C.

Which of the following ECG changes makes the determination of acute MI the most difficult? A) Q wave B) ST-segment elevation C) Left bundle branch block D) First-degree atrioventricular block E) T-wave inversion

C. The changes of left bundle branch block make the determination of an acute MI by an ECG extremely difficult. In these patients, it is particularly important to obtain serum markers of myocardial damage.

The patient described in Question 10.1 (A 68-year-old man with a history of end-stage renal disease is admitted to the hospital for chest pain. On examination, a pericardial friction rub is noted. His ECG shows diffuse ST-segment elevation.) is hospitalized, but there is a delay in initiating treatment. You are called to the bedside because he has become hypotensive with systolic blood pressure of 85/68 mm Hg, a heart rate of 122 bpm, and pulsus paradoxus. A repeat ECG is unchanged from admission. Which of the following is the most appropriate immediate intervention? A) Draw blood cultures and initiate broad-spectrum antibiotics for suspected sepsis. B) Intravenous furosemide for fluid overload. C) Echocardiographic-guided pericardiocentesis. D) Percutaneous coronary intervention for acute myocardial infarction.

C. The clinical picture suggests the patient has developed pericardial tamponade, which may be life threatening and often requires urgent pericardiocentesis.

An obese 50-year-old man with a history of asthma returns with complaints of occasional dyspepsia and nocturnal cough. He wakes up in the morning with a sour taste in his mouth. His current medications include inhaled corticosteroid and a short-acting β2-agonist. Which of the following should be your next step? A) 24-Hour esophageal pH monitoring B) Chest radiograph C) Initiation of omeprazole D) Short course of oral corticosteroids E) Initiation of allergy desensitization

C. The dyspepsia and the sour taste suggest GERD. Aside from acid suppression, other recommendations include dietary modifications and weight reduction. 24-Hour esophageal pH monitoring is indicated only if there is no response to treatment.

A 59-year-old man with a long history of diabetes with chronic renal insufficiency due to diabetic nephropathy is seen in clinic for routine laboratory work. He is asymptomatic, but his glucose is elevated at 258 mg/dL, and his other chemistries are as follows: sodium 135 mEq/L, potassium 5.4 mEq/L, chloride 108 mEq/L, and bicarbonate 18 mEq/L. His creatinine is stable at 2.1 mg/dL. What is the most likely cause of his acidosis? A) Diabetic ketoacidosis B) Lactic acidosis C) Type 4 renal tubular acidosis D) Accidental salicylate overdose

C. The laboratories are consistent with a nonanion gap metabolic acidosis. Patients with chronic kidney disease due to diabetes are prone to subtle volume expansion and low plasma renin activity, leading to hypoaldosteronism. Since aldosterone is the major hormone that promotes potassium excretion, hyperkalemia is the primary electrolyte abnormality. The disorder is typically associated with a mild metabolic acidosis (bicarbonate usually >17 mEq/L). The other illnesses cause anion gap acidosis.

A 70-year-old woman presents for evaluation of a lesion on her left cheek. It has been present for several months. It is slowly enlarging and bleeds if she scratches it. On examination, you find a 7-mm-diameter pearly appearing papule with visible telangiectasias on the surface. Which of the following is the appropriate management of this lesion? A) Close observation and reexamination in 3 months B) Reassurance of the benign nature of the lesion C) Excision D) Local destruction by freezing with liquid nitrogen

C. The lesion is most likely a basal cell carcinoma and should be treated with excision. While the likelihood of metastatic spread is low, these lesions can grow and be locally destructive.

Which of the following is the best treatment for a 39-year-old woman with fever of 103°F, nausea, flank pain, and more than 105 CFU/mL of E coli in a urine culture? A) Oral trimethoprim-sulfamethoxazole for 3 days B) Single-dose ciprofloxacin C) Intravenous and then oral levofloxacin for 14 days D) Oral ampicillin for 21 to 28 days

C. The patient in this scenario has symptoms of an upper urinary tract infection, for example, pyelonephritis, and is moderately ill with nausea. She will need a 14-day course of treatment and may not be able to take oral antibiotics initially, so hospitalization and treatment with intravenous antibiotics likely will be necessary. Single-dose and 3-day regimens are useful only for acute uncomplicated cystitis in women. E coli is frequently resistant to ampicillin.

A 57-year-old man is noted to have a blood pressure of 68/50 mm Hg, heart rate of 140 bpm, elevated jugular venous pressure, inspiratory crackles on examination, and cold clammy extremities. Which of the following is the most likely etiology? A) Septic shock B) Adrenal crisis C) Cardiogenic shock D) Hypovolemic shock

C. The patient is hypotensive with signs of left and right heart failure, that is, probably cardiogenic shock. Septic shock and adrenal crisis both are forms of distributive shock that would produce warm extremities. Hypovolemic shock should have flat neck veins and no pulmonary edema.

A 62-year-old man who works at an automobile assembly line has noticed that he feels pain, fatigue, and numbness in his right arm while working for the last several months. This morning at work, he noticed vertigo, then light-headedness, then lost consciousness for a few seconds. The blood pressure in his right arm is 30 mm Hg lower than that in his left arm. What is the most likely diagnosis? A) Left middle cerebral artery stroke B) Lacunar infarction involving right internal capsule C) Stenosis of right subclavian artery due to atherosclerosis D) Multiple sclerosis

C. The patient likely has subclavian steal: phenomenon of flow reversal in the vertebral artery ipsilateral to a hemodynamically significant stenosis of the subclavian artery. The neurologic symptoms can be caused by vertebrobasilar ischemia.

A 39-year-old man develops a moderate free-flowing pleural effusion following a left lower lobe pneumonia. Thoracentesis reveals straw-colored fluid with gram-positive diplococci on Gram stain, pH 6.9, glucose 32 mg/dL, and LDH 1890. Which of the following is the best next step? A) Send the fluid for culture. B) Continue treatment with antibiotics for pneumococcal infection. C) Tube thoracostomy to drain the effusion. D) Schedule a follow-up chest x-ray in 2 weeks to document resolution of the effusion.

C. The positive Gram stain, low pH, low glucose, and markedly elevated LDH all suggest that this parapneumonic effusion is "complicated," that is, it is unlikely to resolve with antibiotic therapy and is likely to produce loculated pockets of pus, which will require drainage with tube thoracostomy.

A 35-year-old man with no history of cardiac or other vascular disease asks how often he should have routine cholesterol screening. Which of the following is the best answer? A) Every 3 months B) Annually C) Every 5 years D) Every 7 to 10 years

C. The recommended interval for cholesterol screening in this population of healthy adults is every 5 years. Cholesterol levels do not change rapidly over a person's lifetime. A rapid change should prompt investigation for an underlying secondary cause.

A 75-year-old man is noted to have chest pain with exertion and has been passing out recently. On examination he is noted to have a harsh systolic murmur. Which of the following is the best therapy for his condition? A) Coronary artery bypass B) Angioplasty C) Valve replacement D) Carotid endarterectomy

C. The symptoms of aortic stenosis classically progress through angina, syncope, and, finally, congestive heart failure, which has the worse prognosis for survival. This patient's systolic murmur is consistent with aortic stenosis. An evaluation should include echocardiography to confirm the diagnosis, and then aortic valve replacement.

A 67-year-old woman has diabetes and mild hypertension. She is noted to have some diabetic retinopathy, and she states that she cannot feel her legs. She has recurrent episodes of light-headedness when she gets up in the morning. She comes in now because she had fainted this morning. Which of the following is the most likely cause of her syncope? A) Carotid sinus hypersensitivity B) Pulmonary embolism C) Autonomic neuropathy D) Critical aortic stenosis

C. This diabetic patient has evidence of microvascular disease, including peripheral neuropathy, and likely has autonomic dysfunction. Although this is the most likely etiology, one must be concerned about a possible cardiac issue, since the patient had numerous cardiovascular risk factors.

An 18-year-old marathon runner has been training during the summer. He is brought to the emergency room disoriented after collapsing on the track. His temperature is 102°F. A Foley catheter is placed and reveals reddish urine with 3+ blood on dipstick and no cells seen microscopically. Which of the following is the most likely explanation for his urine? A) Underlying renal disease B) Prerenal azotemia C) Myoglobinuria D) Glomerulonephritis

C. This individual is suffering from heat exhaustion, which can lead to rhabdomyolysis and release of myoglobin. Myoglobinuria leads to a reddish appearance and positive urine dipstick reaction for blood, but microscopic analysis of the urine likely will demonstrate no red cells.

An 80-year-old woman is brought to the emergency room (ER) with altered mental status and fever. She is awake and cooperative but is not oriented to time or place. Her blood pressure is normal, her pulse is normal, and her temperature is 101°F. She is found to have pneumonia. Laboratory testing reveals a sodium level of 130 mEq/L but otherwise normal electrolytes. Which of the following is the most appropriate treatment for her? A) IV antibiotic only B) IV antibiotic and aggressive rehydration with IV normal saline C) IV antibiotic and fluid restriction D) IV antibiotic and IV 3% saline

C. This is a common presentation of SIADH due to pneumonia. This patient is euvolemic, so aggressive rehydration is not necessary. Treatment of the underlying pneumonia is key, so antibiotics must be given. With a sodium level of 130 mg/dL, the use of 3% saline is not necessary. The electrolyte abnormality should correct with treatment of the pneumonia and with fluid restriction.

An 85-year-old nursing home resident with a history of congestive heart failure has dementia such that she requires assistance in all activities of daily life. She has a 3-day history of fever and productive cough. Chest x-ray reveals a right middle lobe consolidation. Which of the following is the most appropriate initial antibiotic choice? A) Oral amoxicillin B) Intravenous linezolid C) Intravenous cefepime D) Oral azithromycin

C. This nursing home resident would be considered to have a nosocomial rather than community-acquired infection, with a higher incidence of gram-negative infection. Her age and comorbid medical conditions place her at high risk, requiring hospitalization for intravenous antibiotics such as a third-generation cephalosporin.

A previously healthy 75-year-old man experienced a TIA 2 weeks ago. On physical examination, he is found to have a right carotid bruit. Duplex ultrasound demonstrates a 75% stenosis of the right carotid artery. Which of the following is the best therapy? A) Aspirin plus clopidogrel B) Warfarin (Coumadin) C) Carotid endarterectomy D) Carotid artery stenting

C. This patient has symptomatic carotid disease which include symptoms such as TIAs, episodes of monocular blindness, and small, nondisabling ischemic strokes. A patient who presents with symptomatic carotid disease and stenosis between 70-99% should receive a carotid endarterectomy. It has been found that patients over the age of 70 fare better with a carotid endarterectomy rather than stenting alone. When symptomatic patients present with stenosis between 50-69%, management depends on gender. Women should receive optimal medical management where men should receive a carotid endarterectomy.

An 8-month-old boy presents to the pediatrician in mid-January with 2 days of cough and nasal congestion. His mother reports decreased breastfeeding over the last day and difficulty breathing. The infant's temperature is 38.2°C (100.8°F), heart rate 120/minute, respiratory rate 34/minute, and oxygen saturation 95% on room air. Physical exam is remarkable for intercostal retractions. What exam finding would further suggest the suspected diagnosis? A. Hepatomegaly B. Heart murmur C. Bilateral expiratory wheezing D. Clubbing of the finger E. Diffuse maculopapular rash

C. This patient likely has bronchiolitis, a lower respiratory tract infection affecting the bronchioles that primarily occurs in children under age 2 in the winter months. The most common etiology is respiratory syncytial virus (RSV). In addition to tachypnea and retractions, patients also present with expiratory wheezing and crackles. Hepatomegaly and heart murmur are associated with congenital heart disease and heart failure. Clubbing of the fingers is seen with prolonged pulmonary disease in older children and adults. A diffuse maculopapular rash is a common feature of viral infections in children.

A 48-year-old type 2 Caucasian diabetic man has had persistent blood pressure readings of 150/95 mm Hg for the past 6 months. Current medications include glyburide and metformin. His last HbA1c was 7.9% and the patient has a BMI of 24. On physical examination, position sense is intact but a peripheral neuropathy is detected in a stocking and glove pattern. Vibratory sensation is decreased bilaterally on both lower extremities. Eye examination shows mild papilledema but no cotton wool spots. When questioned, he says that he still occasionally sneaks a cookie after dinner and drinks alcohol nightly. Which of the following is the most appropriate treatment for him? A) DASH diet and recheck blood pressure in 3 months B) Thiazide diuretic alone C) Angiotensin-converting enzyme inhibitor alone D) Combination of angiotensin-converting enzyme inhibitor and thiazide diuretic

C. This patient's blood pressure goal is less than 140/90 mm Hg. He is above this goal, so an ACE inhibitor or ARB is first-line therapy regardless of BMI or HbA1c. The dose of the medication can be maximized if blood pressure is not controlled after 1 month, or another agent can be added.

An 82-year-old woman is admitted to the hospital for altered mental status. Her family says that she has been confused and falling asleep frequently and that she has been hallucinating—talking to people who are not in the room. They report that prior to this illness, she was independent and "sharp as a tack." On urine analysis, she is found to have a urinary tract infection (UTI). Which of the following is the most appropriate treatment? A) Start rivastigmine (Exelon) for worsening of Alzheimer dementia. B) Start an alerting agent such as modafinil (Provigil) for symptomatic treatment of her hypersomnia. C) Start an antibiotic for treatment of her infection and optimize management of any other medical conditions. D) Give her a dose of ziprasidone (Geodon) for her hallucinations.

C. This scenario is one that is commonly seen in elderly patients and is consistent with delirium. The patient is elderly and has an infection, causing both an acute change in her mental status and a fluctuating level of consciousness. The treatment is to treat the underlying infection and any associated medical conditions.

A 43-year-old man presents with headaches that he has had daily for several months. Every morning at work, usually between 9 and 10 am, he has to take 650 mg of acetaminophen to relieve the headache. This has been going on for the past 3 months and he is at the point of looking for a new job, as he thinks that job stress is the cause of his symptoms. His examination is normal. Which of the following is the most appropriate advice for him? A) Continue with the as-needed acetaminophen and find a less-stressful career. B) He should start an antidepressant for headache prophylaxis. C) His headaches are most likely to improve if he stops taking the acetaminophen. D) A triptan is a more appropriate treatment for him.

C. This situation is typical of a medication-related headache. While finding a new, less-stressful job may be beneficial, the problem will not resolve until he discontinues the daily use of his over-the-counter analgesic.

Which of the following distinguishes hyperthyroidism from thyroid storm? A) Tachycardia to heart rate 120 bpm B) Weight loss C) Fever and delirium D) Large goiter

C. Thyroid storm is an exaggeration of hyperthyroid features with extreme tachycardia (heart rate >140 bpm), fever, and central nervous system dysfunction, such as confusion or coma. It is a medical emergency with a high mortality.

An 18-year-old man is noted to be in diabetic ketoacidosis with pH 7.20 and serum glucose level 400 mg/dL. Which of the following is the most accurate statement regarding this patient's potassium status? A) Likely to have a potassium level less than 3 mEq/L. B) Likely to have a potassium level more than 5 mEq/L. C) Likely to have a total body potassium deficit regardless of the serum level. D) Serum level is likely to increase with correction of the acidosis.

C. Total body potassium usually is depleted regardless of the serum level, due to the extracellular shift of potassium.

A 70-year-old man is having difficulty hearing his family members' conversations. He is diagnosed with presbycusis. Which of the following statements regarding his condition is most accurate? A) Presbycusis does not respond to hearing aid use. B) Presbycusis is usually caused by a conductive disorder. C) Presbycusis usually results in loss of speech discrimination. D) Presbycusis usually results in unilateral hearing loss. E) Presbycusis usually results in low-frequency hearing loss.

C. Up to one-third of people older than age 65 suffer from hearing loss. Presbycusis typically presents with symmetric high-frequency hearing loss. There is loss of speech discrimination, so that patients complain of difficulty understanding rapid speech, foreign accents, and conversation in noisy areas. The mechanism is sensorineural rather than a conductive problem.

A 48-year-old woman presents for a well-woman examination. She notes that she had a supracervical hysterectomy in the past. Your records reveal that she had her uterus removed, but the cervix and ovaries were left in place. You also note that she has had Pap smears with HPV cotesting every 5 years since her 20s and that all were normal. She read on the internet that women who have had a hysterectomy no longer need Pap smears. Which of the following would be your advice? A) "You no longer need to get Pap smears since you have had a hysterectomy." B) "You should continue to have Pap smears every 3 years since your hysterectomy is an indication to shorten the interval for testing." C) "You should continue to have Pap smears with HPV cotesting every 5 years since your hysterectomy does not exclude you from routine screening recommendation for your age group." D) "You should continue with annual Pap smears until the age of 50. If they are all normal, you can stop having them at that time."

C. Women who have had a hysterectomy with removal of the cervix for benign indications can discontinue Pap smear screening. Women who still have a cervix should continue with screening for cervical cancer as per screening recommendation for their age group. Cervical cancer screening can be discontinued at age 65, if she has had adequate screening for the last 10 years, which is defined as three consecutive normal Pap smears or two consecutive normal cotesting, with the most recent one being in the last 5 years.

What would NOT be part of the work-up when you suspect a patient with temporal arteritis? A) ESR B) Temporal artery biopsy C) C-Reactive-Protein (CRP) D) HgA1C

D.

A 52-year-old man comes into the outpatient clinic for an annual "checkup." He is in good health, and has a relatively unremarkable family history. He has never smoked cigarettes. For which of the following disorders should a screening test be performed? A) Prostate cancer B) Lung cancer C) Abdominal aortic aneurysm D) Colon cancer

D. Colon cancer screening is given a Level A recommendation by the USPSTF and is routinely offered or provided to all adults older than 50 years. There is insufficient evidence to recommend for or against routine lung or prostate cancer screening. Abdominal aortic aneurysm screening is recommended in men aged 65 to 75 years who have smoked.

A 45-year-old man was admitted for acute pancreatitis, thought to be a result of blunt abdominal trauma. After 3 months he still has epigastric pain but is able to eat solid food. His amylase level is elevated at 260 IU/L. Which of the following is the most likely diagnosis? A. Recurrent pancreatitis B. Diverticulitis C. Peptic ulcer disease D. Pancreatic pseudocyst

D. A pancreatic pseudocyst has a clinical presentation of abdominal pain and mass and persistent hyperamylasemia in a patient with prior pancreatitis.

An 18-year-old woman is brought to the emergency center because she fainted at a rock concert. She apparently recovered spontaneously, did not exhibit any seizure activity, and has no medical history. Her heart rate is 90 bpm and blood pressure 110/70 mm Hg. The neurologic examination is normal. The pregnancy test is negative, and an ECG shows normal sinus rhythm. Which of the following is the most appropriate management? A) Admit to hospital for cardiac evaluation. B) Obtain an outpatient echocardiogram. C) Use 24-hour Holter monitor. D) Reassure the patient and discharge home.

D. A young patient without a medical history, without a seizure activity, and with a history suggestive of emotionally mediated vasovagal syncope has an excellent prognosis.

A 30-year-old woman with no past medical history presents with a productive cough of 2-week duration. She states she also has a runny nose, body aches, congestion, and fevers for the past week. In office she is normotensive, with a normal pulse, and temperature of 101.2°F. Her physical examination is significant for sinus tenderness, boggy nasal turbinates, and crackles in the left lower lobe lung fields. Which one of the following is the best initial step in management? A) Reassure the patient that she likely has a viral infection and it will resolve on its own. B) Order a rapid strep test and treat if positive C) Prescribe amoxicillin for a likely bacterial infection D) Order chest x-ray to rule out possible pneumonia

D. Acute bronchitis is a diagnosis of exclusion in the absence of clinical or radiographic findings concerning for pneumonia. In this patient with fevers, productive cough, and rales on lung examination, it is important to rule out pneumonia. If there is a strong clinical suspicion of community-acquired pneumonia, a chest x-ray is not necessary, and outpatient treatment with antibiotics can be initiated. The diagnosis of streptococcal pharyngitis is made with rapid strep test or throat culture and the decision to order these in office is guided by modified Centor criteria based on the following factors: age, presence of tonsillar exudates, fever, absence of cough.

A 66-year-old man with known metastatic squamous cell carcinoma of the esophagus is brought to the emergency room for increasing lethargy and confusion. He is clinically dehydrated, his serum calcium level is 14 mg/dL, and his creatinine level is 2.5 mg/dL but 1 month ago it was 0.9 mg/dL. Which therapy for his hypercalcemia should be instituted first? A) Intravenous bisphosphonate B) Intravenous furosemide C) Glucocorticoids D) Intravenous normal saline E) Chemotherapy for squamous cell carcinoma

D. Although all of the other therapies listed may be helpful in the treatment of hypercalcemia, given the clinical findings of dehydration and elevated creatinine level with a history of previously normal renal function, volume expansion with normal saline would correct the dehydration and presumed prerenal azotemia, allowing the kidneys to more efficiently excrete calcium. Other therapies can be added if the response to normal saline alone is insufficient.

A 45-year-old man presents with 3 days of watery diarrhea and abdominal cramping. He has no sick contacts and has not traveled recently. He is not currently taking any medications, but he was prescribed amoxicillin 2 weeks ago for a sinus infection. Which of the following tests is most likely to identify the cause of his diarrhea? A) Stool guaiac B) Evaluation of stool for fecal leukocytes C) Evaluation of stool for ova and parasites D) C difficile toxin immunoassay

D. Although any antibiotic can cause C difficile colitis, clindamycin, cephalosporins, and penicillins are the most commonly implicated

A 56-year-old man is brought into the emergency room intoxicated with alcohol. He has repeated bouts of emesis and is found choking. Lung examination reveals some crackles in the right lung base. Which of the following is the most appropriate management? A) Initiate azithromycin. B) Initiate corticosteroid therapy. C) Initiate haloperidol therapy. D) Observation with follow-up chest radiograph.

D. Antibiotic therapy is generally not indicated for aspiration pneumonitis, but patients need to be observed for clinical deterioration. Azithromycin would be appropriate to treat an atypical pneumonia such as Mycoplasma. If this was an aspiration pneumonia as evidenced by fever, purulent sputum production and a patient known to aspirate antibiotic treatment with clindamycin would be appropriate.

Which of the following counseling strategies is most likely to enhance your patients' smoking cessation rates? A) Discuss smoking cessation techniques only with patients who ask for your advice, as others will resent your suggestions. B) Emphasize primarily the health risks of smoking. C) Note in each patient's chart that you have discussed cessation, so that you don't repeat the message to the same patient at subsequent visits. D) Ask about smoking cessation at each encounter.

D. Asking patients about tobacco use is a key to promoting cessation. It is important to ask each patient at each visit and to be prepared to provide advice and assistance at anytime.

A 17-year-old adolescent boy presents to the ER with a temperature of 101.0°F (38.3°C), a deep nonproductive cough, and generalized malaise for 3 days. He doesn't recall being around any particular sick contacts but is around many people in his after-school job in sales and at school. He states that he never had the chicken pox and is unaware of what immunizations he received as a child. He was diagnosed at age 12 with leukemia but has since been healthy. He is worried that his cancer may no longer be in remission. A chest x-ray reveals bilateral, diffuse infiltrates. Which of the following is the most likely cause of illness? A) Pneumonia caused by S pneumoniae B) Pneumonia caused by P jiroveci C) Pneumonia caused by L pneumophila D) Pneumonia caused by M pneumoniae E) Pneumonia caused by H influenzae

D. Bilateral, diffuse infiltrates are more likely to be seen in patients with pneumonia caused by atypical agents, such as Mycoplasma, than in patients with typical pneumonia or aspiration pneumonia. Legionella, another atypical pneumonia, usually is in older patients and the patient did not have diarrhea. It is more likely that the patient contracted an atypical pneumonia than having a relapse of leukemia with such profound immunodeficiency as to have contracted a Pneumocystis infection, with no prior symptoms.

A 59-year-old diabetic woman had suffered an acute anterior wall MI. Five days later, she gets into an argument with her husband and complains of chest pain. Her initial ECG shows no ischemic changes, but serum cardiac troponin I levels are drawn and return mildly elevated at this time. Which of the following is the best next step? A) Use thrombolytic therapy. B) Treat with percutaneous coronary intervention. C) Perform coronary artery bypass. D) Perform serial ECGs and obtain CK-MB. E) Prepare the patient for dialysis.

D. Diabetic patients can have myocardial ischemia or infarction with atypical or absent symptoms. Clinical suspicion and a liberal use of cardiac enzymes are required. Troponin levels often remain elevated for 7 to 10 days and should not be used to diagnose reinfarction, especially if the levels are trending downward. New ECG findings or rapidly rising markers such as serum myoglobin or CK-MB can be used in this setting.

You see a 70-year-old woman in your office for a routine checkup, and you order a DEXA scan for bone mineral density screening. The T score returns as -2.5 SD in the spine and -2.6 in the hip. Which of the following statements is most accurate? A) This patient has osteopenia. B) Estrogen replacement therapy should be started with an anticipated rebuilding of bone mass to near-normal within 1 year. C) Swimming will help build bone mass. D) Bisphosphonates would reduce the risk of hip fracture by 30%-50%.

D. Estrogen primarily inhibits loss of bone mass, although it can help to build a modest amount of bone mass, but also may be associated with increased thrombotic and cardiovascular risk. Weight-bearing exercise, not swimming or bicycling, is important in preventing osteoporosis. Bisphosphonates decrease the incidence of hip fractures by 30% to 50%.

A 22-year-old man complains of acute hemoptysis over the past week. He denies smoking or pulmonary disease. His blood pressure is 130/70 mm Hg, and his physical examination is normal. His urinalysis also shows microscopic hematuria and red blood cell casts. Which of the following is the most likely etiology? A) Metastatic renal cell carcinoma to the lungs B) Acute tuberculosis of the kidneys and lungs C) Systemic lupus erythematosus D) Goodpasture disease (antiglomerular basement membrane)

D. Goodpasture (antiglomerular basement membrane) disease typically affects young males, who present with hemoptysis and hematuria. Antibody against type IV collagen, expressed in the pulmonary alveolar and glomerular basement membrane, leads to the pulmonary and renal manifestations. Wegener granulomatosis typically affects older adults, and includes more systemic symptoms such as arthralgias, myalgias, and sinonasal symptoms, and these patients are positive for ANCA.

A 30-year-old woman who is 12 weeks' postpartum is noted to have adrenal insufficiency and a very distinct tan, although she hardly ventures outside. Which of the following is the most likely etiology? A) Long-term steroid use B) Sheehan syndrome (pituitary insufficiency) C) Brain tumor D) Autoimmune adrenal destruction

D. Hyperpigmentation occurs as a result of increased melanocyte-stimulating factor, a byproduct of ACTH, and occurs in primary adrenal insufficiency. Secondary causes of adrenal insufficiency such as Sheehan syndrome result in low ACTH levels and do not cause the "tanned" appearance. A brain tumor such as an ACTH producing pituitary adenoma would cause over activation of the adrenal glands resulting in a Cushingoid appearance and hyperpigmentation due to the excess ACTH.

A 48-year-old man with no significant medical history and no symptoms is found to have elevated cholesterol at a health screening. Which of the following tests is part of the routine evaluation of this problem? A) ECG B) Stress test C) Complete blood count (CBC) D) Thyroid-stimulating hormone

D. Hypothyroidism is a potential cause of secondary dyslipidemia. A TSH is a reasonable test to perform in this setting. There is no indication to screen for CHD with an ECG or stress test in this asymptomatic person. Other tests to perform could include fasting blood glucose, liver enzymes, and a measurement of renal function.

A 43-year-old man who is an alcoholic is admitted to the hospital with acute pancreatitis. He is given intravenous hydration and is placed NPO. Which of the following findings is a predictor of higher mortality? A) His age B) Initial serum glucose level of 60 mg/dL C) BUN of 18 mg/dL D) Disorientation, with Glasgow Coma Scale score of 10 E) Amylase level of 1000 IU/L

D. Impaired mental status is a poor prognostic sign. Other findings associated with higher mortality include a BUN > or = to 25, presence of SIRS, age over 60 and presence of a pleural effusion. Notably, the amylase level does not correlate to the severity of the disease.

A 66-year-old Caucasian woman has an average blood pressure of 155/70 mm Hg despite appropriate lifestyle modification efforts. Her only other medical problems are osteopenia, kidney stones, and mild depression. Her last lipid panel revealed a total cholesterol of 160 mg/dL, high-density lipoprotein (HDL) 40 mg/dL, and low-density lipoprotein (LDL) 90 mg/dL. Which of the following would be the most appropriate treatment at this time? A) Lisinopril (Prinivil, Zestril) B) Propranolol (Inderal) C) Amlodipine (Norvasc) D) Chlorthalidone E) Losartan (Cozaar)

D. In the JNC 8 guidelines, calcium channel blockers, thiazides, ARBs, and ACE inhibitors are first line in nonblack patients over age 60. In this case, β-blockers may worsen the depression. Thiazide diuretics may improve osteoporosis, and reduce hypercalciuria which can reduce nephrolithiasis.

Which of the following is the most common cause of secondary adrenal insufficiency? A) Autoimmune process B) Surgical excision C) Hemorrhagic shock D) Exogenous corticosteroids E) ACTH failure due to panhypopituitarism

D. Long-term steroid use, with secondary suppression of pituitary secretion of ACTH, is the most common cause of secondary adrenal insufficiency. Autoimmune adrenalitis is the most common cause of primary adrenal insufficiency. Surgical excision of the adrenal glands would result in primary adrenal insufficiency. Hemorrhage of the adrenal glands is more common in the setting of sepsis and another cause of primary adrenal insufficiency.

A 67-year-old man with dizziness and a positive stool guaiac test. A) Normal MMA; decreased serum folate level B) Elevated MMA; decreased serum B12 level C) Elevated ferritin; normal MCV; decreased serum iron level D) Decreased ferritin; decreased MCV; decreased serum iron level

D. Low serum iron, low MCV, and low ferritin levels, along with a finding of blood in the stool, are consistent with iron-deficiency anemia. A workup for the source of the GI blood loss should ensue.

Which of the following is the best next step for a patient with acoustic neuroma? A) Prescription for a selective serotonin reuptake inhibitor B) Referral for a hearing aid C) Lumbar puncture and serology for syphilis D) Referral for an MRI E) Reassurance

D. MRI is the diagnostic test of choice.

A 44-year-old woman has a 5-month history of malaise and stiff hands in the morning that improve as the day goes by. She notes that both hands are involved at the wrists. Initial laboratory tests show an elevated ESR and high positive anti-CCP. Which of the following treatments is most likely to lead to the best long-term disease outcome for this patient? A) Allopurinol B) Ibuprofen C) Naproxen D) Methotrexate E) Intravenous ceftriaxone

D. Morning stiffness, involvement of the hands, and symmetric arthritis are common features of RA. According to the ACR/EULAR, this patient meets the criteria for the diagnosis of newly presenting RA in that she has joint involvement, positive serology, elevated acute-phase reactants, and duration of symptoms more than 6 weeks. DMARD therapy, such as the use of methotrexate, would be indicated. Methotrexate as a disease-modifying agent would alter the natural history of the disease rather than just treat the symptoms.

One year ago, a 24-year-old woman had an episode of diplopia of 2 weeks' duration. The symptoms resolved completely. Currently, she complains of left arm weakness but no headache. Which of the following is the most likely diagnosis? A) Recurrent transient ischemic attacks B) Subarachnoid hemorrhage C) Complicated migraine D) Multiple sclerosis

D. Multiple neurologic deficits separated in space and time in a young patient are suggestive of multiple sclerosis. Symptoms lasting longer than 24 hours as well as the patient's age makes transient ischemic attacks (even if recurrent) less likely to be the cause of her symptoms. A subarachnoid hemorrhage will often present with a "thunder clap headache" or "the worst headache" of the patient's life and is usually an isolated event. A complicated migraine can include symptoms such as changes in vision and arm weakness but with or before the onset of the headache.

What postexposure prophylaxis should the student getting stuck with needle from a hep B& C positive patient receive? A) HBIg B) Oral lamivudine C) Intravenous immunoglobulin (IVIG) D) Reassurance

D. No postexposure prophylaxis is definitively indicated. The student has detectable protective antibody levels against the hepatitis B virus, and if the levels are judged to be adequate, the student is protected against infection. Oral lamivudine is a treatment for chronic hepatitis B infection and is part of an antiretroviral prophylaxis if the patient was HIV positive. There is no effective prophylaxis for hepatitis C exposure.

Which of the following is most accurate? A) Cough caused by captopril may resolve with switching to enalapril. B) Initial treatment of a chronic cough should include codeine or a similar opiate derivative to suppress the cough. C) Cough caused by reflux can be effectively ruled out by a negative history of heartburn or dyspepsia. D) More than one condition is often responsible for causing chronic cough in a given patient.

D. Often more than one condition is responsible for causing chronic cough in a given patient. Cough from ACE inhibitors is class dependent, and change to another class of antihypertensives is more appropriate. The etiology of chronic cough should be determined prior to suppression of the cough because treatment of the underlying condition is the most effective approach. A patient with GERD may present with the sole manifestation of cough, or it may present "silently."

Penicillins inhibit which of the following bacterial processes/compounds? A) Protein synthesis B) Topoisomerase C) Dihydropteroate synthase D) Cell-wall synthesis

D. Penicillins inhibit synthesis of the bacterial cell wall. Chloramphenicol, tetracyclines, macrolides, ketolides, oxazolidinones, aminoglycosides, spectinomycin, and the lincomycin bind to bacterial ribosomes to inhibit protein synthesis. The fluoroquinolones inhibit activity of bacterial topoisomerase to inhibit protein synthesis, and the sulfonamides inhibit bacterial dihydropteroate synthase to block folic acid synthesis and cell growth.

A patient with which of the following conditions requires antimicrobial prophylaxis before dental surgery? A) Atrial septal defect B) Mitral valve prolapse without mitral regurgitation C) Previous coronary artery bypass graft D) Previous infective endocarditis

D. Prior endocarditis damages valvular surfaces, and these patients are at increased risk for reinfection during a transient bacteremia, as may occur during dental procedures or some other GI or genitourinary tract procedures. All of the other conditions mentioned have a negligible risk of endocarditis, the same as in the general population, and antibiotic prophylaxis is not recommended by the American Heart Association.

Which one of the following patients presenting with chest pain is at the highest risk for an acute myocardial infarction? A) A 40-year-old woman on proton pump inhibitor for reflux disease B) A 75-year-old man with parasternal chest pain, lipid abnormalities, and no past history or cardiac disease C) A 23-year-old man recently diagnosed with hypertrophic cardiomyopathy D) A 67-year-old man with a history of a prior angioplasty, with chest pain radiating to the neck and complaint of diaphoresis

D. Risk factors for increased likelihood of acute MI are male gender, age older than 60 years, chest pain radiating to neck, jaw, arm, or shoulder, and a prior history of angina or acute MI.

Several friends develop vomiting and diarrhea 6 hours after eating food at a private party. Which of the following is the most likely etiology of the symptoms? A) Rotavirus B) Giardia C) E coli D) S aureus E) Cryptosporidium

D. S aureus toxin usually causes vomiting and diarrhea within a few hours of food ingestion.

A 35-year-old man presents to your clinic with progressive jaundice, pruritis, and fatigue. He has ulcerative colitis. Choose the cause that is probably responsible for the patient's presentation. A) Wilson disease B) Hematochromatosis C) Primary biliary cirrhosis D) Sclerosing cholangitis E) Autoimmune hepatitis F) Alcohol-induced hepatitis G) Viral hepatitis

D. Sclerosing cholangitis is an autoimmune destruction of both the intrahepatic and extrahepatic bile ducts and often is associated with inflammatory bowel disease, most commonly ulcerative colitis. Patients present with jaundice or symptoms of biliary obstruction; cholangiography reveals the characteristic beading of the bile ducts.

A 56-year-old Asian woman with history of hypertension, diabetes, and newly diagnosed polycystic kidney disease presents for follow-up for hypertension. Routine laboratory work shows elevated calcium of 13 mg/dL and an elevated phosphate level. The patient denies weight loss, is taking only metoprolol for her blood pressure, and denies recent history of immobilization. Given as above, which etiology of hypercalcemia would you be most concerned about in this patient? A) Primary hyperparathyroidism as this is the most common etiology of hypercalcemia B) Iatrogenic hypercalcemia secondary to medications C) A primary vitamin D deficiency given her age D) Secondary hyperparathyroidism due to renal disease

D. Secondary hyperparathyroidism occurs in patients with early renal disease when due to hyperphosphatemia, hypocalcemia, and impaired production of 1,25-dihydroxyvitamin D by the failing kidneys, PTH levels increase abnormally, causing in turn elevated levels of calcium. In this disorder therefore you would see increased PTH and increased calcium, signaling a disruption of the normal feedback cycle.

A 45-year-old man has hypertension. A thiazide diuretic agent had been prescribed with continued elevated blood pressure. The inclusion of spironolactone to the thiazide diuretic is done to achieve which of the following? A) Reduce hyperuricemia B) Reduce Mg+ loss C) Decrease the loss of Na+ D) Reduce K+ loss

D. Spironolactone is a "potassium-sparing" diuretic that reduces K+ excretion in the collecting duct. It diminishes the K+-wasting effects of thiazide diuretics.

68-year-old male with hypertension presents for annual examination. On review of systems he reports urinary hesitancy and nocturia. Your examination reveals a nontender but enlarged prostate without nodules. On review of his blood pressure logs and clinic readings he is averaging values of 150/80 mm Hg. Which of the following medications would offer treatment of the hypertension and prostatic symptoms? A) Furosemide B) Aliskiren C) Propranolol D) Terazosin

D. Terazosin is an α1-adrenoreceptor antagonist that is a useful antihypertensive. It also reduces symptoms associated with benign prostatic hyperplasia. Patient should be cautioned on side effects of orthostatic hypotension. The goal is to start low and go slow with the dosing to minimize side effects. Furosemide, a loop diuretic, would improve the blood pressure but worsen the nocturia.

A 34-year-old man of Mediterranean descent with a family history of anemia (MCV Ferritin TIBC RDW) A) Increased Decreased Increased Decreased B) Decreased Decreased Increased Increased C) Normal Increased Normal Normal D) Decreased Increased Normal Normal E) Increased Increased Decreased Increased

D. Thalassemia usually leads to a microcytic anemia with uniform red cell size (normal RDW) and excess iron stores.

A 55-year-old man is noted by his family members to be forgetful and become disoriented. He has difficulty making it to the bathroom in time and complains of feeling as though "he is walking like he was drunk." Which of the following therapies is most likely to improve his condition? A) Intravenous penicillin for 21 days B) Rivastigmine C) Treatment with fluoxetine for 9 to 12 months D) Ventriculoperitoneal shunt E) Enrollment into Alcoholics Anonymous

D. The classic triad for normal pressure hydrocephalus is dementia, incontinence, and gait disturbance; one treatment is shunting the cerebrospinal fluid.

A 25-year-old man presents with a 2-week history of low-grade fever, slight cough, malaise, and myalgias, and is noted on physical examination to have enlarged posterior cervical lymph nodes and significant splenomegaly. His CBC shows a lymphocytosis with ALC 10,000/μL, normal hemoglobin level, and normal platelet count. The peripheral smear shows large atypical lymphocytes. What is the most likely diagnosis? A) ALL B) CLL C) Acute HIV infection D) EBV infection E) Pertussis

D. The clinical presentation of fever, malaise, adenopathy, and splenomegaly is consistent with infectious mononucleosis, which is most often associated with EBV, but can also be due to CMV or other viral infections. The absence of cytopenias makes ALL unlikely. The lymphocytosis in CLL and pertussis consist of mature small lymphocytes. Acute HIV infection can present similarly to mononucleosis, but does not typically cause massive splenomegaly.

An 18-year-old male patient is noted to have motor tics and involuntary, obscene vocalizations. Which of the following medications is indicated in the treatment of this disorder? A) Trihexyphenidyl B) Phenytoin C) Carbamazepine D) Haloperidol E) Levodopa

D. The clinical scenario described is associated with Tourette syndrome. A variety of drugs may help suppress the tics that are characteristic of this syndrome. These include haloperidol, pimozide, trifluoperazine, and fluphenazine. Antiepileptics such as carbamazepine and phenytoin are not useful. Levodopa is the drug of choice in treating advanced Parkinson disease. Trihexyphenidyl and benztropine are useful in suppressing the parkinsonism that may develop with haloperidol administration, but are not useful in the management of Tourette syndrome.

A 9-year-old girl is being seen in your office with fever and difficulty breathing. You are concerned about the diagnosis of epiglottitis. Which of the following is the most accurate statement regarding epiglottitis? A) Child usually prefers to be in prone position. B) Radiographic finding of "steeple sign." C) Every effort should be made to visualize the epiglottis in the office to confirm the diagnosis. D) Diagnosis is decreasing in incidence.

D. The incidence of epiglottitis has markedly reduced since the introduction of the Hib vaccine. Children with epiglottitis are more likely to be in the tripod position than prone. The "steeple sign" is seen in croup; the "thumb" sign is seen in epiglottitis. Visualization of the epiglottis should preferentially occur in an operating room, where immediate intubation or tracheostomy can occur.

A 45-year-old African-American woman presents for a routine examination. You notice a 9-mm-diameter lesion on the palm of her right hand that is dark black, slightly raised, and has a notched border. When asked about it, she says that it has been present for about a year and is growing. A friend told her not to be concerned because, "black people don't get skin cancer." Which of the following is your advice? A) Her friend is correct and this is nothing to worry about. B) While anyone can get skin cancer, this lesion has primarily benign features and can be safely observed. C) This lesion is suspicious for cancer but this is most likely a metastasis from another source, such as a breast cancer. D) This lesion is suspicious for a primary melanoma and needs further evaluation immediately.

D. The lesion described is suspicious for an acral lentiginous melanoma and needs evaluation. While skin cancers are more common in persons with lighter skin, they can occur in persons with any skin color or tone.

A 72-year-old man, with a long history of hypertension, presents to the emergency department (ED) complaining of a 2-day history of emesis and 36 hours of no urination. On examination, the abdomen is firm and tender, and the prostate is enlarged. His serum creatinine level is 3.4 mg/dL. Which the following is the best next step? A) Give him IV fluids and see if he begins to make urine. B) Perform a renal ultrasound in the ED. C) Maintain tight control of his blood pressure. D) Place an indwelling Foley catheter.

D. The patient has an enlarged prostate that has caused urinary obstruction and potentially reversible renal failure, depending on at which point the obstruction is resolved. Placing the Foley catheter will usually allow for significant reversal of an elevated Cr. Following catheter placement, the urine output needs to be carefully monitored and the Cr repeated later. Another clue is the tense lower abdomen that is caused by a very enlarged bladder. It is especially important to rely on clinical examination skills in elderly patients who have less-than-optimal communication skills as a consequence of dementia or who have a history of stroke when evaluating for a cause.

A 45-year-old man with no significant past medical history presents with severe back pain after lifting heavy boxes at work 2 days ago. Other than his back pain, his review of symptoms is negative. The pain radiates from his lower back down his right posterior thigh to his great toe when you perform both a straight leg raise and the contralateral leg raise tests. His strength, sensation, and reflexes are intact and symmetrical. Which of the following imaging studies should be done first in the evaluation of this patient? A) Plain radiographs B) MRI C) Computed tomography (CT) scan D) No imaging indicated E) Bone scan

D. The patient has signs and symptoms of a herniated disc. There is no evidence that imaging within the first month has any morbidity benefit.

A 49-year-old man with a long-standing history of chronic renal failure as a consequence of diabetic nephropathy is brought to the emergency room for nausea, lethargy, and confusion. His physical examination is significant for an elevated jugular venous pressure, clear lung fields, and harsh systolic and diastolic sounds heard over the precordium. Serum chemistries reveal K 5.1 mEq/L, CO2 17 mEq/L, BUN 145 mg/dL, and creatinine 9.8 mg/dL. Which of the following is the most appropriate therapy? A) Administer IV insulin and glucose. B) Administer IV sodium bicarbonate. C) Administer IV furosemide. D) Urgent hemodialysis.

D. The patient has uremia, hyperkalemia, and (likely) uremic pericarditis, which may progress to life-threatening cardiac tamponade unless the underlying renal failure is treated with dialysis. As for the other treatments, insulin plus glucose would treat hyperkalemia, and bicarbonate would help with both metabolic acidosis and hyperkalemia, but in this patient, his potassium and bicarbonate levels are only mildly abnormal and are not immediately life threatening. Furosemide will not help because he does not have pulmonary edema and has renal insufficiency.

A 65-year-old man who has a prosthetic heart valve is hospitalized for a knee replacement surgery, and placed on intravenous (IV) heparin for anticoagulation before the procedure. He drinks one glass of wine each weekend and has been diagnosed with osteoarthritis for which he takes acetaminophen. His platelet count was normal, but now is 32,000/mm3. Which of the following is the most likely cause of the thrombocytopenia? A) Prosthetic heart valve B) Alcohol intake C) Acetaminophen D) Heparin

D. The patient likely has heparin-induced thrombocytopenia, which may be confirmed by assay for HIT antibodies, including IgG against platelet factor 4. Treatment consists of stopping the heparin, and starting a direct Xa inhibitor such as argatroban or bivalirudin.

A 42-year-old man with polycystic kidney disease who complained of a sudden onset of severe headache and then lost consciousness A) Migraine headache B) Tension headache C) Cluster headache D) Subarachnoid hemorrhage E) Meningitis

D. The sudden onset of severe headache with diminution in level of consciousness is classic for subarachnoid hemorrhage. This patient likely had rupture of a cerebral artery aneurysm, which is associated with polycystic kidney disease.

A 59-year-old woman has been placed on warfarin (Coumadin) after being found to have had chronic atrial fibrillation. She is noted to have an INR of 5.8, is asymptomatic, and has no overt bleeding. Which of the following is the best management for this patient? A) Transfuse with erythrocytes. B) Give vitamin K. C) Give fresh frozen plasma. D) Hold warfarin.

D. The target INR with warfarin is 2 to 3; thus, 5.8 is markedly elevated. However, because she has no overt bleeding and is asymptomatic, holding the warfarin until the INR reaches the acceptable range is a reasonable approach. Patients with overt bleeding require more urgent intervention such as clotting factors.

A 28-year-old woman complains of excessive bleeding from her gums and has petechiae. Her CBC shows a platelet count of 22,000/mm3 with a hemoglobin of 8.9 g/dL and a WBC count of 87,000/mm3. Which of the following is the most likely etiology of her low platelet count? A) Immune thrombocytopenia purpura B) Systemic lupus erythematosus C) Drug-induced thrombocytopenia D) Acute leukemia

D. The thrombocytopenia is seen with other hematologic abnormalities, the most abnormal of which is a markedly elevated WBC count, suggesting acute leukemia.

A 28-year-old man presents for evaluation of headaches. He has had several episodes of unilateral throbbing headaches that last 8 to 12 hours. When they occur, he gets nauseated and just wants to go to bed. Usually they are relieved after he lays down in a dark, quiet room for the remainder of the day. He is missing significant work time due to the headaches. He has a normal examination today. Which of the following statements is accurate regarding this situation? A) He needs a CT scan of his head to evaluate for the cause of his headache. B) When he gets his next headache, he should breathe in 100% oxygen and use a triptan medication. C) If he has not already done so, he should use aspirin 650 mg orally every 4 hours as needed and take a stress-management class. D) An injectable or nasal spray triptan is most appropriate.

D. This patient gives a history very consistent with common migraine headaches. There are no red flags found on history or examination, so no further testing is necessary at this point. As he has significant nausea, he may benefit from nonoral medication. A triptan delivered by injection or nasal spray is a reasonable starting point for him.

Ms N is a widowed 80-year-old woman who comes to your office for a yearly physical examination. She is doing well and has no complaints. A routine ECG is performed and reveals atrial fibrillation with a heart rate of 75 bpm. Blood pressure is similar to her last visit, which is 128/76 mm Hg. She does not have a history of diabetes, heart failure, or stroke. Her current medications include mirtazapine for treatment of depression. What is the appropriate management of this patient? A) Recommend one 325 mg aspirin per day for antithrombic therapy. B) Repeat ECG in 3 months. If atrial fibrillation is still present, begin adjusted-dose warfarin therapy. C) Immediately begin low molecular weight heparin injections and start warfarin therapy. D) Begin adjusted dose warfarin therapy.

D. This patient has a CHADS2-VASc score of 3 due to her age of 80 years (2 points) and being female (1 point). Based on this score, recommended anticoagulation therapy is adjusted-dose warfarin with a target INR goal of 2.0 to 3.0. Aspirin (A) is only the therapy of choice for patients <65 years old with no other risk factors (HTN, DM, CHF, TIA/stroke). Repeating ECG (B) is not indicated. Heparin (C) therapy is only indicated in hemodynamically unstable patients who are undergoing immediate cardioversion, either pharmacologic or electrical.

A 45-year-old woman is noted to have dizziness, pounding of the chest, and fatigue of 3 hours' duration. On examination, she is noted to have a blood pressure (BP) of 110/70 mm Hg and heart rate of 180 bpm. She is noted on ECG to have atrial fibrillation, and a prior baseline ECG showed delta waves. The ED physician counsels the patient regarding cardioversion, but the patient declines. Which of the following is the best therapy for her condition? A) Digoxin B) Angiotensin-converting enzyme (ACE) inhibitor C) Calcium channel blocker D) Procainamide

D. This patient has atrial fibrillation but with WPW as indicated by the delta wave. In this setting, the typical agents used to treat atrial fibrillation that slow the AV node are contraindicated since the conduction through the accessory pathway could actually accelerate. DC cardioversion is an option; however in a hemodynamically stable patient, procainamide may be used since it will slow propagation through the accessory pathway. Because this patient declines cardioversion, procainamide is the best choice.

A 58-year-old man presents to his physician for follow-up of his hypertension and hyperlipidemia. He also reports chest pain and feeling short of breath after climbing two flights of stairs or walking three to four blocks. The symptoms resolve after several minutes of rest. Which of the following drugs is contraindicated as a first-line agent in the treatment of this patient's new condition? A) Labetalol B) Nitroglycerin C) Enalapril D) Nifedipine E) Aspirin

D. This patient has new onset of angina. Rapid release, short-acting dihydropyridines (nifedipine) are contraindicated because they increased mortality in multiple trials. β-Blocking agents are the agents of choice since they increase survival; nitroglycerin helps to abate chest pain, but has not been shown to impact survival.

A 48-year-old man presents for follow-up of an elevated calcium level of 12.3 mg/dL found on routine screening laboratory tests at his last well-man visit. He takes no medications other than an occasional antihistamine for allergies. He recently started smoking a half-pack of cigarettes per day. He was prompted to attend to his well-man visit by his wife who claims that he has become forgetful, has a decreased appetite, and has had a 10-lb weight loss over the past 2 months. As part of his follow-up laboratory tests, you obtain a serum PTH, which comes back within the normal range. Which of the following is the next step in diagnosis? A) Chest x-ray B) Repeat calcium after hydration C) Measurement of PTH-rP levels D) Measurement of urinary calcium excretion

D. This patient has symptomatic hypercalcemia. He has an inappropriately normal PTH level, which should be suppressed with this degree of hypercalcemia. The next step is to measure a 24-hour urinary calcium excretion to determine if this condition represents primary hyperparathyroidism (most common) or familial hypocalciuric hypercalcemia (rare).

A 12-year-old girl was brought to the emergency department because of severe sore throat, muffled voice, drooling, and fatigue. She had been sick for the past 3 days and is unable to eat because of painful swallowing. The parents deny any history of recurrent pharyngitis. The patient still managed to open her mouth and you were able to see an abscess at the upper pole of the right tonsil with deviation of the uvula toward the midline. Examination of the neck reveals enlarged and tender lymph nodes. Which of the following is the most appropriate management? A) Analgesics for pain B) Oral antibiotics C) Nebulized racemic epinephrine D) Incision and drainage of the abscess E) Tonsillectomy and adenoidectomy

D. This patient is suffering from peritonsillar abscess. Of the choices listed, incision and drainage is the most appropriate. Tonsillectomy is only indicated if there are confirmed cases of recurrent pharyngitis and peritonsillar abscess.

A 56-year-old woman admits to a 60-pack-year smoking history. She complains of fatigue and dyspnea with minimal exertion, and a cough that is productive each morning. Which of the following is the most likely finding in this patient? A) Higher diffusing capacity of lung for carbon monoxide (DLCO) B) Decreased residual volume C) Normal to slightly increased FEV1 D) Decreased FEV1/FVC E) Decreased FVC

D. This patient likely has COPD, based on the smoking history and symptoms. A decrease in the FEV1/FVC ratio is the hallmark of airflow obstruction. The FEV1 is decreased in obstructive, as well as in restrictive, lung disease. The diffusing capacity is typically deceased in COPD as well as intrinsic restrictive lung disease. The DLCO indicates the adequacy of the alveolar-capillary membrane; the residual volume is the volume of air remaining in the lungs after a maximal expiratory effort and is usually increased in COPD due to air trapping.

A 76-year-old widowed man who lives alone presents to clinic with increasing shortness of breath and chest pain at rest for the past 2 weeks. He has had chronic hypertension and coronary artery disease (CAD) for 20 years for which he takes hydrochlorothiazide (HCTZ), enalapril, and aspirin 81 mg daily. Other medical problems include hyperlipidemia, peripheral vascular disease, and gastroesophageal reflux disease (GERD) which are controlled by lovastatin, warfarin, and omeprazole. Two years ago, he suffered a cerebrovascular accident that was localized to the brain stem. He now has dysphagia and is noted to cough frequently at night. He has no cough at present and has not been able to take his temperature at home. Which of the following is the best next step? A) Upper endoscopy B) Removal of angiotensin-converting enzyme (ACE) inhibitor C) Nitroglycerine patch D) Chest radiograph

D. This patient most likely has aspiration pneumonia. With impairment of the gag reflex after cerebrovascular accident (CVA), he is more likely to aspirate during sleep, indicated by his cough. His GERD is well controlled by medication, so upper endoscopy is not warranted at this time. Nitroglycerine patches may be indicated if he described symptoms more related to angina. An ACE inhibitor would cause a cough unrelated to the time of day.

A 45-year-old man is noted to have a blood pressure of 80/40 mm Hg, heart rate of 142 bpm, and fever of 102°F. His abdomen is tender, particularly in the right lower quadrant, and acute appendicitis is diagnosed. Three liters of 0.9% saline are infused and intravenous antibiotics are administered as he is prepared for surgery. His blood pressure falls to 70/42 mm Hg. Which of the following is the most appropriate next step? A) Administer a beta-blocker to control his heart rate. B) Check a cortisol level and administer corticosteroids. C) Infuse fresh frozen plasma (FFP). D) Initiate norepinephrine intravenous infusion. E) IV morphine for pain control.

D. When septic shock is refractory to volume resuscitation with at least 30 cc isotonic fluid per kilogram ideal body weight administration, then addition of intravenous norepinephrine is the next step. Corticosteroids can be administered empirically if hypotension is refractory to vasopressors. Intravenous morphine might lower his blood pressure further. FFP is used when the patient shows evidence of coagulopathy such as disseminated intravascular coagulation.

A 40-year-old man with no significant past medical history presents with a 2-day history of alternating fever and rigors, diaphoresis, fatigue, and a productive cough. He admits to mucoid sputum of moderate quantities. He denies a history of smoking, alcohol use, recent travel, or sick contacts. He further denies chest pain, palpitations, hemoptysis, rashes, abdominal pain, nausea, vomiting, or diarrhea. On physical exam, he is found to be tachypneic and was observed to be intermittently coughing. The pulmonary exam was notable for bronchial breath sounds over the right anterior fourth, fifth, and sixth intercostal spaces. A chest radiograph revealed a right middle lobe consolidation. What additional physical exam finding would be consistent with this patient's most likely diagnosis? A. Cheyne-Stokes respiration B. Bradycardia C. Decreased tactile fremitus D. Decreased whispered pectoriloquy E. Dullness to percussion

E.

Which of the following is the most accurate statement regarding H pylori infection? A) It is more common in North America than in the developing world. B) It is associated with the development of colon cancer. C) Eradication of H pylori eliminates most cases of nonulcer dyspepsia. D) The route of transmission is believed to be sexually transmitted. E) It is a cause of both duodenal and gastric ulcers.

E. Although H pylori is clearly linked to gastric and duodenal ulcers and probably to gastric carcinoma and lymphoma, whether it is more common in patients with nonulcer dyspepsia is unclear. Less than 10% of patients with nonulcer dyspepsia improve after H pylori treatment. H pylori infection is more common in developing countries.

A 7-month-old infant was brought by her mother to an outpatient clinic because of a 2-day history of fever, copious nasal secretions, and wheezing. The mother volunteered that the baby has been healthy and has not had these symptoms in the past. The infant's temperature is noted to be 100.7°F (38.1°C), her respiratory rate is 50 breaths/min, and her pulse oximetry is 95% on room air. Physical examination reveals no signs of dehydration, but wheezing is heard on bilateral lung fields on auscultation. The infant shows no improvement after three treatments with nebulized albuterol. Which of the following is the recommended treatment? A) Continued nebulized albuterol every 4 hours. B) Antihistamines and decongestants. C) Antibiotics for 7 days. D) Initiate Synagis. E) Supportive care with hydration and humidified oxygen.

E. Bronchiolitis is the most likely diagnosis in this case. There is no established treatment for bronchiolitis except for supportive management of the patient's symptoms. Because the infant did not respond to an albuterol trial, there is no justification for continuing its use. Antihistamines, decongestants, and antibiotics are not effective. Synagis is not helpful in the acute setting.

A 54-year-old woman is noted to have cervical cancer and presents with significant vaginal bleeding with a hemoglobin level of 7 g/dL. Her left leg is swollen, which on Doppler investigation reveals a deep venous thrombosis. Which of the following is the best treatment for the thrombus? A) Intravenous unfractionated heparin B) Fractionated subcutaneous heparin C) Subcutaneous unfractionated heparin D) Oral warfarin (Coumadin) E) Vena cava filter

E. Cervical cancer with significant vaginal bleeding is a relative contraindication for anticoagulation. Thus, a vena cava filter is the most appropriate choice in this patient.

Which of the following therapies is most likely to provide the greatest benefit to a patient with chronic stable emphysema and a resting oxygen saturation of 86%? A) Inhaled tiotropium daily B) Inhaled albuterol as needed C) Oral prednisone daily D) Supplemental oxygen used at night E) Supplemental oxygen used continuously

E. For patients with chronic hypoxemia, supplemental oxygen has a significant impact on mortality, with a greater benefit with continuous usage, rather than intermittent or nocturnal-only usage. Bronchodilators such as tiotropium and albuterol improve symptoms and FEV1, but offer no mortality benefit. Chronic use of oral corticosteroids should be avoided because of unfavorable side effects such as osteoporosis, glucose intolerance, and gastrointestinal (GI) side effects.

A 34-year-old man with a past history of asthma presents to an acute care clinic with an asthma exacerbation. Treatment with nebulized albuterol and ipratropium does not offer significant improvement, and he is then admitted to the hospital. He is afebrile, has a respiratory rate of 24 breaths/min, pulse rate is 96 beats/min, and oxygen saturation is 93% on room air. On examination, he has diffuse bilateral inspiratory and expiratory wheezes, mild intercostal retractions, and a clear productive cough. Which one of the following should be the next step in the management of this patient? A) Chest physical therapy B) Inhaled corticosteroids C) Azithromycin orally D) Theophylline orally E) Oral corticosteroids

E. Hospital management of acute exacerbations of asthma should include inhaled short-acting bronchodilators and systemic corticosteroids. The efficacy of oral versus intravenous corticosteroids has been shown to be equivalent. Antibiotics are not needed in the treatment of asthma exacerbations unless there are signs of infection. Inhaled ipratropium is recommended for treatment in the emergency department, but not in the hospital. Chest physical therapy and theophylline are not recommended for acute asthma exacerbations.

A 15-year-old adolescent girl has elevated liver enzymes and a positive antinuclear antibody (ANA). Choose the one cause (A-G) that is probably responsible for the patient's presentation. A) Wilson disease B) Hematochromatosis C) Primary biliary cirrhosis D) Sclerosing cholangitis E) Autoimmune hepatitis F) Alcohol-induced hepatitis G) Viral hepatitis

E. Idiopathic or autoimmune hepatitis is a less-well-understood cause of hepatitis that seems to be caused by autoimmune cell-mediated damage to hepatocytes. A subgroup of these patients includes young women with positive ANAs and hypergammaglobulinemia who may have other symptoms and signs of systemic lupus erythematosus.

Which is not a common symptom of Giant cell arteritis? A) Acute vision loss or amaurosis fugax B) Jaw claudication C) Fatigue D) Headache E) Weight Gain

E. Most patients with vasculitis loose rather than gain weight due to their systemic illness.

A 45-year-old woman comes in for follow-up of an ankle sprain that occurred while she was jogging 2 weeks ago. X-rays done at your initial visit were negative for fracture. She has been unable to run because of persistent stiffness. She is frustrated as she states she has tried everything including ice, NSAIDs, bracing, and weight bearing as tolerated. Examination reveals no joint instability or focal tenderness. Which of the following is the most appropriate management at this time? A) Referral for therapeutic ultrasound as this has shown to benefit patients who have failed conservative treatment. B) Referral to chiropractor for soft-tissue techniques to loosen up the stiff muscles. C Change to another NSAID as the current one may not be strong enough. D Urge immobilization of the ankle as she may be doing too much too soon. E Discuss importance of rehabilitative stretching, strengthening, and range-of-motion exercises to gain functionality.

E. The most common cause of a stiff or painful joint following a sprain is inadequate rehabilitation. At 2 weeks postinjury, the patient is considered to be out of the acute phase and the focus should be on regaining range of motion and strength instead of NSAIDs, immobilization, or alternative treatments (choices A, B, or C). Studies have not shown any benefit with therapeutic ultrasound or hyperbaric oxygen therapy for the treatment of acute ankle sprains.

A 22-year-old college student with fever, headache, photophobia, and 25 white blood cells per high-power field, but no red blood cells or xanthochromia in CSF A) Migraine headache B) Tension headache C) Cluster headache D) Subarachnoid hemorrhage E) Meningitis

E. The presence of white blood cells but no red blood cells in the CSF is indicative of meningeal inflammation, likely due to viral or bacterial infection.

A 65-year-old dialysis patient is found to have a serum potassium level of 6.8 mEq/L, which is verified on a STAT repeat level. An ECG shows peaked T waves and a widened QRS complex. What is the first intervention that should be made at this point? A) IV glucose and insulin administration B) Arrangement for a dialysis treatment C) Oral Kayexalate given D) IV furosemide E) IV calcium

E. This patient has hyperkalemia with cardiac changes—an acute, life-threatening condition. The first intervention should be to give IV calcium to stabilize the cardiac membranes and reduce the risk of arrhythmia. After this, interventions can be made to lower the potassium level.

A 13-year-old adolescent boy has a nonproductive cough and mild shortness of breath on a daily basis. He is awakened by the cough at least five nights per month. Which one of the following would be the most appropriate treatment for this patient? A) A long-acting β-agonist daily B) A short-acting β-agonist daily C) Oral prednisone daily D) An oral leukotriene inhibitor as needed E) Inhaled corticosteroids daily

E. This patient has moderate persistent asthma. The most effective treatment is daily inhaled corticosteroids. A leukotriene inhibitor would be less effective and as a controller should be used daily. Oral prednisone daily is problematic due to the risk of adrenal insufficiency. Short- and long-acting β-agonists are not recommended as daily therapy because they are considered rescue medications rather than asthma controllers.

A 35-year-old morbidly obese woman returns to clinic with sudden onset of night sweats, chills, shortness of breath, and cough productive of yellowish-green sputum. Her vital signs show a temperature of 104.0°F, with a respiratory rate of 30 breaths/min, heart rate of 100 beats/min, pulse oximetry is 93% on room air. She was seen 8 days ago for headache, fever of 102.0°F, nonproductive cough, and myalgias. She was prescribed a dose of oseltamivir for 5 days. She felt better after taking the medication initially but now feels she is getting worse. She is sent to the emergency room for expedited evaluation. Assuming admission for pneumonia, which of the following is the best empiric antibiotic treatment for this patient? A) A 14-day trial of oseltamivir B) Azithromycin C) Penicillin D) Levofloxacin E) Ceftriaxone with vancomycin

E. This patient is most likely suffering from postinfluenza pneumonia. Due to the higher risk of mortality associated with morbid obesity and some vital signs indicating risk of sepsis, this patient should be evaluated quickly for possible admission. If admitted for pneumonia, antibiotic coverage should cover for Pneumococcus and S aureus. Levofloxacin would be reasonable for community-acquired pneumonia but does not provide good coverage for staph infections.

A 23-year-old Caucasian woman presents to your office for a routine well-woman examination. She feels well without any concern/complaints. She is a non-smoker with no significant past medical history. Her last Pap smear with cotesting was last year. Her last tetanus shot was 2 years ago and she has received a flu shot for the current season already. She states that she has never been checked for skin cancer before and asks to be checked for it today. She denies excessive sun exposure and has never been to a tanning facility before. Based on the above information, which preventive services does she need at today's visit? A) Pap smear and tetanus shot B) Whole body examination to check for skin lesions based on age and race C) Tetanus shot only D) Mammogram and tetanus shot E) Counseling to avoid excessive sun exposure and tanning booths

E. This patient is up-to-date on her cervical cancer screening and immunizations for her age. She is not yet of the age where screening mammography would be recommended. Per USPSTF recommendations, whole body skin cancer screening is not generally recommended but counseling to reduce the risk of development of skin cancer would be advised.

A 64-year-old woman with a history of hypertension and angina pectoris presents with chest pain for the last 3 hours. She describes the pain as "sharp," it is worse when she inhales deeply, and it is not relieved by sublingual nitroglycerin. Her ECG shows ST elevation in most leads. Which of the following is the most likely diagnosis in this patient? A) Unstable angina pectoris B) Myocardial infarction C) Aortic dissection D) Congestive heart failure E) Pericarditis

E. This patient likely has pericarditis. The pain is described as sharp in nature rather than dull, aching, pressure. The pain is exacerbated by inspiration, and finally there is global ST-segment elevation noted on the ECG.

A young woman eats raw seafood and 2 days later develops fever, abdominal cramping, and watery diarrhea. Which of the following is the most likely etiology of the symptoms? A) Rotavirus B) Giardia C) E coli D) S aureus E) Vibrio

E. Vibrio is a common cause of diarrhea among people who eat raw seafood.

A 40-year-old woman undergoing chemotherapy develops fevers and respiratory symptoms. A chest radiograph is ordered; it reveals bilateral fluffy pulmonary infiltrates. A bronchoscopy with biopsy is performed; the specimen is found to contain septate hyphae with acute-single branching. What treatment should the patient receive? A. Erythromycin B. Tetracycline C. Penicillin D. Vancomycin E. Voriconazole

E. Voriconazole. The patient's symptoms are most indicative of a pneumonia-like infection, most likely stemming from the fungus Aspergillus fumigatus. Voriconazole is considered first-line treatment for invasive aspergillosis. It is available in both oral and intravenous forms and has good bioavailability. It is hepatically cleared and has less toxicity compared to amphotericin B, which is also used to treat aspergillosis. Vancomycin would not be used for a fungal infection. It is used to treat methicillin-resistant Staphylococcus aureus (MRSA), Clostridium difficile, and enterococcal disease. Erythromycin is not used to treat fungal infections. Erythromycin is a macrolide and was formerly the drug of choice for the treatment of Legionella, but now azithromycin is preferred. Tetracycline is used for some "unusual" infections such as Mycoplasma, Ureaplasma, nocardiosis, plague, rocky mountain spotted fever, scrub typhus, and Q fever. Penicillin is most active against Streptococcus spp., Neisseria gonorrhoeae, Treponema pallidum, and Listeria spp. It is used clinically to treat syphilis, Streptococcus pyogenes pharyngitis, and as a prophylactic agent in rheumatic heart disease.

What type of ocular finding would be less likely due to a chemical burn? A) Conjunctival whiting B) Corneal Scarring C) Cataract D) Increased ocular pressure E) Retinal detachment

E. Without a history of trauma a chemical burn would not necessarily affect the retina.

A 28-year-old woman has been told she has rheumatic heart disease, specifically mitral stenosis. Which of the following murmurs is most likely present? A) Diastolic rumble at apex of the heart B) Early diastolic decrescendo at right upper sternal border C) Holosystolic murmur at apex D) Late-peaking systolic murmur at right upper sternal border

A. A diastolic rumble at the cardiac apex suggests mitral stenosis. The early diastolic decrescendo murmur is typical of aortic regurgitation; holosystolic murmur at the apex is typical of mitral regurgitation; and late-peaking systolic murmur at the upper sternal border is typical of aortic stenosis.

A 66-year-old woman is brought in by her family because of difficulty with memory and disorientation that has worsened over the past 6 months. A careful history and physical examination is performed. Which of the following tests is most appropriate in this patient? A) Head CT or MRI B) Lumbar puncture C) Rapid plasma reagin (RPR) D) Electroencephalogram (EEG)

A. A noncontrast head CT or MRI is recommended by the American Academy of Neurology for the routine evaluation of dementia. All of the other tests may be appropriate if there is a finding on the history or examination that calls for further testing (an exposure to syphilis, episodes suggestive of seizures, or symptoms of normal pressure hydrocephalus for which a spinal tap may be performed).

A 30-year-old woman takes prednisone 15 mg/d for systemic lupus erythematosus. She is admitted to the hospital for a cholecystectomy. Which of the following is the most important intervention for her? A) Hydrocortisone intravenously before surgery and every 6 hours for 24 hours. B) Double the prednisone the night before and hold her steroids the day of the surgery. C) Use of cyclophosphamide in lieu of corticosteroids for 2 weeks following surgery to promote wound healing. D) Cancel the surgery and use lithotripsy to break up the stones.

A. A stress dose of corticosteroids is important to prevent adrenal insufficiency before surgery.

A 45-year-old woman with type 2 diabetes presents to the clinic with decreased vision in the left eye for 1 year, 1+ proteinuria, a baseline Cr of 1.6 mg/dL, an low-density lipoprotein (LDL) of 135 mg/dL, blood pressure of 145/92 mm Hg, and occasional chest pain for the past 2 months. Which of the following is the best medication to start the patient on at this time? A) ACE inhibitor B) β-Blocker C) Oral nitrate D) Thiazide diuretic

A. ACE inhibitors would help in hypertension treatment and to protect renal function in this patient. Both diabetes and CKD are known to be cardiovascular risk equivalents. Other factors, such as uncontrolled blood pressure and cholesterol, add to the patient's high risk, which is why it is so important for all diabetics and CKD patients to improve all modifiable risk factors. The goals become much more stringent when looking at these two groups of patients.

A 42-year-old woman is noted to have Type II diabetes for 20 years. She is noted to have hypertension with BP in the 150/94 range. The urinalysis shows mild proteinuria. Which of the following drugs would be the best to treat the hypertension in this individual? A) Enalapril B) Propranolol C) Hydrochlorothiazide D) Nifedipine

A. ACE inhibitors, such as enalapril, have been shown to reduce the progressive loss of renal function that is often seen in diabetic patients. The nonselective beta blocker, propranolol, would worsen the diabetes.

A 68-year-old man is found to have an incidental finding of anemia while in the hospital for alcohol abuse. A) Normal MMA; decreased serum folate level B) Elevated MMA; decreased serum B12 level C) Elevated ferritin; normal MCV; decreased serum iron level D) Decreased ferritin; decreased MCV; decreased serum iron level

A. Alcohol abuse is a common cause of folate deficiency. A normal MMA level essentially rules out a concomitant vitamin B12 deficiency.

A 68-year-old man is found to have an incidental finding of anemia while hospitalized with pneumonia. His physical examination is normal except for crackles in the left lower lobe. Serum laboratory examinations reveal a normal MMA and a decreased serum folate level. Which of the following is the best next step? A) Administer CAGE questionnaire B) Esophagogastroduodenoscopy C) Serum iron assay D) Neurology consultation

A. Alcohol abuse, which may be assessed by the CAGE questionnaire, is a common cause of folate deficiency. CAGE is an acronym which stands for Cut back, Annoyed, Guilty, and Eye-opener. A normal MMA level essentially rules out a concomitant vitamin B12 deficiency. Gastric endoscopy—to look for atrophic gastritis—would be indicated for pernicious anemia. A serum iron assay would likely be high because of increased turnover of iron in patients with megaloblastic anemia due to either B12 or folate deficiency. A neurology consultation would be needed if the patient had neurologic signs or symptoms of B12 deficiency.

A 55-year-old man is noted to have moderately severe congestive heart failure with impaired systolic function. Which of the following drugs would most likely lower his risk of mortality? A) Angiotensin-converting enzyme inhibitors B) Loop diuretics C) Digoxin D) Aspirin

A. Angiotensin-converting enzyme inhibitors and beta-blockers decrease the risk of mortality for patients who have CHF with impaired systolic function. For this reason, these agents are the initial therapies of choice to treat CHF. They both prevent and can even, in some circumstances, reverse the cardiac remodeling.

A 62-year-old asymptomatic woman is noted to have multiple myeloma and hypercalcemia, but no bone lesions or end-organ damage. Which of the following therapies is useful for immediate treatment of the hypercalcemia? A) Bisphosphonates. B) Erythropoietin. C) Dexamethasone plus thalidomide. D) Interferon-alpha. E) Observe without treatment since she is asymptomatic.

A. Bisphosphonates are helpful in controlling hypercalcemia through inhibition of osteoclastic bone reabsorption. Dexamethasone, in combination with thalidomide, is useful in treatment of the myeloma, with a slower effect on the calcium level. Erythropoietin is inappropriate and is used to increase synthesis of red blood cells in those with renal failure. Interferon-alpha is also not an appropriate treatment for hypercalcemia. It is appropriate to treat chronic hepatitis B and as an adjuvant to surgical treatment for malignant melanoma.

A 22-year-old African-American woman presents with worsening cough over 6 weeks, which did not improve with a course of antibiotics or antitussives. Her serum calcium level is found to be 12.5 mg/dL, and a chest x-ray reveals bilateral hilar lymphadenopathy. She has erythema nodosum on her legs. Which of the following is the most likely diagnosis? A) Sarcoidosis B) Mycoplasma pneumonia C) Acute lymphoblastic leukemia D) Squamous cell carcinoma of the lung E) Pulmonary embolism

A. Both sarcoidosis and lymphoma can present with cough, dyspnea, and hilar adenopathy on chest x-ray. In approximately 10% of cases, sarcoidosis can cause elevated calcium levels through the production of 1,25-vitamin D that occurs in the macrophages of the granulomas. This can also be seen in granulomas caused by tuberculosis and in lymphoma. Leukemia usually does not present in this manner, although it can cause hypercalcemia. Squamous cell carcinoma of the lung would be unusual in a patient of this age, and the radiographic presentation is atypical. The case scenario is consistent with Lofgren syndrome, an acute presentation of sarcoidosis, which includes hilar adenopathy, erythema nodosum, migratory polyarthralgia, and fever, seen most often in women.

A pregnant woman who smokes 1 pack of cigarettes a day asks for your advice regarding smoking cessation while she is pregnant. Which of the following statements is most appropriate? A) Bupropion is pregnancy category C. B) Varenicline is pregnancy category B and relatively safe in pregnancy. C) Nicotine gum delivers a lower and safer dose of nicotine than the nasal spray. D) The use of smoking cessation products during pregnancy frequently leads to adverse outcomes.

A. Bupropion and varenicline are both pregnancy category C. Pregnant smokers should be encouraged to quit without the use of any pharmacologic agents. However, pharmacologic aids to increase the rate of smoking cessation during pregnancy can be used, after discussion with the patient of the risks and benefits of the medications and of continued smoking. Cessation of smoking at anytime during the pregnancy is likely to provide health benefits for the mother and fetus. Nicotine gum delivers higher doses of nicotine than its nasal spray counterpart.

Which of the following statements regarding available treatments for smoking cessation is accurate? A) Bupropion can be used in combination with nicotine supplements. B) Nicotine gum is most effective if chewed continuously, to promote a constant release of the nicotine. C) Nicotine supplements are most effective when used as needed for withdrawal symptoms. D) All of the available agents are more effective when used in combinations with each other.

A. Bupropion can be used in combination with any of the nicotine supplementation products. The nicotine products can also be used in combination with each other. Varenicline has not been studied for use with other smoking cessation agents. Two common pitfalls in using nicotine supplementation are using supplementation only when having withdrawal symptoms and failing to use nicotine gum correctly. The gum should be chewed briefly and then parked in the cheek. It is less effective if chewed continuously.

Which of the following is the most likely physical examination findings in a patient with COPD? A) Diffuse expiratory wheezing B) Clubbing of the fingers C) Bibasilar inspiratory crackles with increased jugular venous pressure (JVP) D) Inspiratory stridor E) Third heart sound

A. COPD is characterized by chronic airway obstruction, with most airflow resistance occurring in small airways of the lower respiratory tract, producing expiratory wheezing. Inspiratory stridor would occur with upper airway, usually extrathoracic, obstruction. Clubbing is not generally a feature of COPD and should prompt investigation for another disease process such as a bronchogenic carcinoma. Crackles, elevated JVP, and an S3 are signs of congestive heart failure.

A third-year medical student has been reading about the dangers of excessive anticoagulation and bleeding potential. He reviews the charts of several patients with atrial fibrillation currently taking Coumadin. Which of the following patients is best suited to have anticoagulation discontinued? A) A 45-year-old man who has normal echocardiographic findings and no history of heart disease or hypertension, but a family history of hyperlipidemia B) A 62-year-old man with mild chronic hypertension and dilated left atrium, but normal ejection fraction C) A 75-year-old woman who is in good health except for a prior stroke, from which she has recovered nearly all function D) A 52-year-old man with orthopnea and paroxysmal nocturnal dyspnea

A. Clinical factors associated with a higher risk for embolic stroke include congestive heart failure, hypertension, age >75, diabetes, or prior stroke. Echocardiographic factors include dilated left atrium or the presence of an atrial thrombus. The man in answer A has "lone atrial fibrillation" with a CHADS2 score <2, and has a low risk for stroke and thus would not benefit from anticoagulation.

Which of the following most likely will lead to a nonanion gap acidosis? A) Diarrhea B) Lactic acidosis C) Diabetic ketoacidosis D) Ethylene glycol ingestions

A. Diarrhea leads to bicarbonate loss and usually does not affect the anion gap. All other choices- lactic acidosis, DKA, and ethylene glycol ingestion are causes of a high anion gap metabolic acidosis. Common causes can be remembered with the MUDPILES mneumonic. (Methanol, Uremia, Diabetic ketoacidosis, Propylene glycol, Iron/Isoniazid/Infection, Lactic acidosis, Ethylene glycol, Salicylates).

A 25-year-old white woman who is in training for a competitive marathon complains of "hitting a wall" and "getting short of breath quicker than she should." She complains of coughing at the end of her training runs, and states that she may be expecting too much of herself. She does not smoke, has no significant family history, and no history of occupational or environmental exposures. Her physical findings including lung examination are unremarkable. Spirometry reveals normal values both pre- and post-albuterol treatment. What would be the most reasonable first step in treatment of this patient? A) Trial of albuterol MDI before exercise B) Chest radiograph C) Chest CT D) Counseling for athletic burnout or stress E) An echocardiogram (ECG) to rule out pulmonary hypertension or cardiac disorder

A. Exercise-induced asthma or bronchoconstriction is a common, underdiagnosed condition in athletes. Many of the athletes are unaware of the problem. It is defined as a 10% lowering of forced expiratory volume in 1 second (FEV1) when challenged with exercise. It is much more common in high-ventilation sports and in cold, dry air. The incidence among cross-country skiers is as high as 50%. A physical examination and spirometry at rest will be normal unless there is underlying asthma. Methacholine challenge testing can be ordered, but if it is not available, a trial with an albuterol inhaler is reasonable. Pulmonary or cardiac dysfunction not found during the physical examination is much less likely and, therefore, an ECG and chest x-ray would not be indicated until common etiologies have been ruled out. Psychological causes are also a less likely etiology.

A 74-year-old man with no prior medical problems faints while shaving. He has a quick recovery and has no neurologic deficits. His blood sugar level is normal, and an ECG shows a normal sinus rhythm. Which of the following is the most useful diagnostic test of his probable condition? A) Carotid massage B) Echocardiogram C) Computed tomographic (CT) scan of head D) Serial cardiac enzymes

A. He likely has carotid hypersensitivity; thus, careful carotid massage (after auscultation to ensure no bruits are present) may be given in an attempt to reproduce the symptoms. Carotid massage in an older patient should be used with caution because it may lead to cerebral ischemia, emboli of a plaque, or to atrial fibrillation.

A 33-year-old man is diagnosed with essential hypertension. He is started on a blood pressure medication, and after 6 weeks, he notes fatigue, rash over his face, joint aches, and effusions. A serum antinuclear antibody (ANA) test is positive. Which of the following is the most likely agent? A) Hydralazine B) Propranolol C) Thiazide diuretic D) Nifedipine E) Enalapril

A. Hydralazine is associated with a lupus-like presentation, with photosensitivity, malar rash, joint pain, and sometimes pericardial effusion or pleural effusion.

Which of the following therapies would most likely decrease the number of sickle cell crises? A) Hydroxyurea B) Folate supplementation C) Prophylactic penicillin D) Pneumococcal vaccination

A. Hydroxyurea and decitabine may decrease the incidence of sickle cell crises by increasing levels of hemoglobin F.

Which of the following situations is associated with an increased risk of intimate partner violence? A) Pregnancy B) Older age C) Higher income D) Married status

A. Intimate partner violence can occur in any relationship, but the risk is increased in certain situations, which include young age, low income status, pregnancy, mental illness, alcohol or substance use by victims or partner, separated or divorced status, and a history of childhood sexual/physical abuse.

Which of the following represents the decrement in speech commonly exhibited by the patient with parkinsonism? A) Progressively inaudible speech B) Neologisms C) Expressive aphasia D) Receptive aphasia E) Word salad

A. Language is not disturbed in Parkinson disease, as it is with aphasias. The clarity and volume of speech is what suffers. Handwriting is similarly disturbed, as the patient has increasingly smaller and less legible penmanship as he or she continues to write. This is referred to as micrographia.

A 13-year-old adolescent girl presents with fever and sore throat of 48-hour duration. She has a temperature of 101°F in office and is tachycardic with a pulse of 118 beats/min. Her physical examination is positive for tender, enlarged left cervical lymphadenopathy and tachycardia. Her pharynx is erythematous but without tonsillar enlargement or exudate. She has had no cough. What is the best step in management? A) Treat empirically with antibiotics. B) Order rapid strep test and, if positive, treat with antibiotics. C) Neither further testing nor antibiotics. D) Order throat culture and, if positive, treat with antibiotics.

A. Management of strep pharyngitis is frequently guided by modified Centor criteria, which calculates a probability of strep throat based on a scoring system (presented earlier in the chapter). This patient gets one point for the presence of fever, tender cervical adenopathy, absence of cough, and age. You could reasonably consider an empiric antibiotic treatment for GAS in her.

Which of the following patients is most likely to be a candidate for bone mineral density screening? A) A 65-year-old, thin, white woman who smokes and is 15 years postmenopausal B) A 40-year-old white woman who exercises daily and still menstruates C) A healthy 75-year-old white man who is sedentary D) A 60-year-old overweight African-American woman E) A 35-year-old asthmatic woman who took prednisone 40 mg/d for a 2-week course 1 week ago

A. Of the choices, this woman is the only individual with risk factors. Risk factors include white race, age, postmenopausal status, smoking, positive family history, poor nutritional status, and chronic treatment with a drug known to predispose to bone loss.

Ototoxicity and nephrotoxicity are characteristic adverse effects of which of the following? A) Aminoglycosides B) β-Lactam antibiotics C) Chloramphenicol D) Fluoroquinolones

A. Ototoxicity and nephrotoxicity are characteristic adverse effects of aminoglycosides. Chloramphenicol can cause GI disturbances, reversible suppression of bone marrow, and rarely aplastic anemia. As a group, the β-lactam antibiotics can cause hypersensitivity and have the potential to cause anaphylactic shock. The fluoroquinolones can cause GI disturbances, reversible arthropathy, and arrhythmias.

Which one of the following patients should be promptly referred for endoscopy? A) A 65-year-old man with new onset of epigastric pain and weight loss B) A 32-year-old patient whose symptoms are not relieved with ranitidine C) A 29-year-old H pylori-positive patient with dyspeptic symptoms D) A 49-year-old woman with intermittent right upper quadrant pain following meals

A. Patient in answer A has "red flag" symptoms: he is older than 45 years and has new-onset symptoms. Patient in answer B may benefit from the reassurance of a negative endoscopic examination. Patient in answer C, however, may benefit from treatment of H pylori first. Some studies indicate this approach may be cost-saving overall. This patient could be sent for an endoscopic examination if she does not improve following the therapy.

Osteomyelitis A) Salmonella spp B) S pneumoniae C) Parvovirus B19 D) Fat embolus E) Hematuria

A. Patients with sickle cell disease are at risk for Salmonella osteomyelitis.

A 32-year-old man comes for evaluation of right shoulder pain that he has had for the past 3 weeks. He thinks that he injured it playing softball but does not remember a specific injury. There is no bruising or swelling. He gets pain in the joint on external rotation and abduction, but has preserved range of motion. Which of the following is the initial imaging test of choice? A) X-ray B) MRI C) CT scan D) Arthrogram

A. Plain film x-rays are the diagnostic imaging test of choice for the initial evaluation of the painful joint. In patients who have normal x-rays and who have a suspected soft-tissue (ligament, tendon, or cartilage) injury, MRI scanning is usually the next most appropriate imaging study to perform.

During the winter, a young daycare worker develops watery diarrhea. Which of the following is the most likely etiology of the symptoms? A. Rotavirus B. Giardia C. E coli D. S aureus E. Cryptosporidium

A. Rotavirus is a common etiology for watery diarrhea, especially in the winter.

A 60-year-old woman presents with the results of her DEXA scan. She has a T score of -1.5 SD at the hip and -2.5 at the spine. Which of the following is the most accurate interpretation of these results? A) She has osteoporosis at the spine and osteopenia at the hip. B) She has osteoporosis in both areas. C) This is a normal examination. D) She has osteoporosis of the hip and osteopenia at the spine. E) You need to know the Z score.

A. The T score is the number of standard deviations of a patient's bone mineral density from the mean of young, adult, white women. It is the standard measurement of bone mineral density used by the World Health Organization. Osteopenia is defined as a T score of 1 to -2.4. A score of -2.5 SD is the definition of osteoporosis. A Z score is the number of standard deviations from the mean bone mineral density of women in the same age group as the patient.

A 58-year-old woman comes to your office complaining of fatigue. She has also noticed a burning sensation in her feet over the past 6 months. A CBC shows anemia with an increased MCV. Which of the following is the most likely cause of her anemia? A) Lack of intrinsic factor B) Inadequate dietary folate C) Strict vegetarian diet D) Chronic GI blood loss

A. The clinical presentation and CBC findings are consistent with macrocytic anemia due to B12 deficiency. Pernicious anemia (lack of intrinsic factor) is the most common cause. B12 deficiency can also be seen in patients who follow a strict vegetarian diet; however, the body's B12 stores can last several years before they are depleted.

A 75-year-old woman, diagnosed with stage 0 CLL 1 year ago and being monitored without treatment, now complains of fatigue and dyspnea. She has no palpable adenopathy or splenomegaly, no rashes or arthritis, and her CBC shows ALC 11,000/μL, with hemoglobin 6.8 mg/dL, and platelet count 127,000/μL. What is the most appropriate diagnostic test? A) Direct antiglobulin (Coombs) test B) Antinuclear antibody C) Bone marrow biopsy D) Test for Lewis alloantibody

A. The most likely diagnosis is AIHA (autoimmne hemolytic anemia), which can be confirmed by detection of antibody and/or complement components on the surface of the red blood cell (RBC), usually by the direct antiglobulin (Coombs) test. AIHA is a common complication of CLL. Antinuclear antibody (ANA) to screen for systemic lupus erythematosus (SLE) has a low probability in a woman of this age, without other clinical features of SLE. Bone marrow biopsy to evaluate for bone marrow failure due to CLL could be considered, but rapid progression to stage III/IV would be unlikely. Lewis alloantibodies have no clinical significance.

A third-year medical student is researching various recommendations for the care of the geriatric patient. Which of the following statements is most accurate? A) USPSTF recommends routine screening for colorectal cancer in all adults starting at the age of 50. B) USPSTF recommends stopping screening for cervical cancer with Pap smear in all women past the age of 65. C) The USPSTF recommends that all men should be screened for prostate cancer with prostate-specific antigen (PSA) testing annually starting at the age of 50. D) Herpes zoster vaccination is recommended for all adults over the age of 50.

A. The only accurate answer among the choices is that regarding colorectal screening. There is no recommendation for annual routine PSA testing for prostate cancer screening. Pap smears can be safely discontinued in women over the age of 65 who have had adequate prior screening. The herpes zoster vaccine is recommended for routine use at age 60 or older.

A 22-year-old African-American woman presents with fatigue, arthralgias, and a nagging dry cough for the past 6 weeks, but no shortness of breath. On physical examination, her lungs are clear to auscultation, and she has bilateral pretibial tender erythematous raised nodules. Which of the following is your best next step? A) Chest radiograph B) High-resolution CT C) Empiric treatment for postnasal drip D) Antinuclear antibody E) Initiation of antituberculosis therapy

A. The patient has clinical features suggestive of sarcoidosis given the new cough, arthralgias, and description of erythema nodosum. The initial, most cost-effective study is a chest radiograph. Hilar lymphadenopathy with or without interstitial infiltrates would solidify a diagnosis of sarcoidosis. A high-resolution CT may be ordered if the patient has interstitial lung disease, but it is not the first study of choice. Postnasal drip does not explain the patient's other symptoms. An antinuclear antibody would not necessarily identify the cause of the cough or provide a diagnosis.

A 4-year-old boy accompanied by his mother presents with fever, sore throat, muffled voice, and breathing and swallowing difficulty. The child is leaning forward with his head and nose tilted upward and forward. He is irritable, with moderate respiratory distress and inspiratory stridor. Pulse is 94/min, BP is 110/70 mm Hg, temperature is 101°F. What is the next step to confirm the diagnosis? A. Direct fiberoptic laryngoscopy in operating room B. Indirect laryngoscopy C. Examination with tongue depressor D. Lateral neck radiograph E. Complete blood count and blood culture

A. The patient has symptoms of acute epiglottitis, a diagnosis that can be made on clinical grounds. The next step to confirm the diagnosis is direct fiber-optic laryngoscopy performed in a controlled environment (usually the operation theater) to visualize and culture the edematous larynx and to secure the airway through placement of an endotracheal tube. Direct visualization in the examination room with tongue depressor or indirect laryngoscopy is not recommended due to the high risk of immediate laryngospasm and complete airway obstruction. Lateral neck radiograph usually reveals an enlarged edematous epiglottis (thumbprint sign). Radiographs can be used for diagnosis in stable patients or patients with low suspicion for epiglottitis as a rule-out test. This patient has moderate respiratory distress and inspiratory stridor, so radiographs are not the best option; securing airway is pertinent. Laboratory investigations like complete blood count (CBC) typically reveal elevated leukocytes with neutrophil predominance, and blood cultures are usually positive. These investigations assist the diagnosis but may delay the critical step of placing the endotracheal tube.

A 31-year-old woman with a long history of intermittent severe unilateral throbbing headache lasting hours to days associated with nausea and photophobia, but no preceding symptoms and no visual disturbance, occurring once or twice per month A) Migraine headache B) Tension headache C) Cluster headache D) Subarachnoid hemorrhage E) Meningitis

A. The patient's history is strongly suggestive of migraine, given its unilateral and throbbing character, and the associated symptoms of nausea or photophobia. Most patients with disabling headache have migraine. Tension headache should have none of these features.

A 58-year-old white woman presents complaining of low back pain for exactly 1 month after a fall. She has no history of fever, unexplained weight loss, diabetes, or cancer. Her past medical history is significant for mild persistent asthma and nicotine dependence. She had a hysterectomy for uterine fibroids at age 40. Which of the following characteristics should prompt further evaluation of her pain? A) History of corticosteroid use B) Caucasian ethnicity C) Time course of back pain D) History of cocaine use E) Premenopausal age

A. The patient's history is suspicious for a vertebral compression fracture that could be secondary to osteoporosis. Osteoporosis commonly develops in postmenopausal women, and can occur in patients who have received corticosteroid therapy. The time course of her pain is 4 weeks; 6 weeks and greater is a "red flag" symptom for further evaluation with radiographic imaging. While osteoporosis is more common in Caucasian women, it is not considered a "red flag." Postmenopausal women are at greater risk for osteoporosis rather than premenopausal women. Smoking and alcohol dependence are risk factors for osteoporosis; there is no evidence that cocaine use contributes to the development of osteoporosis.

During which of the following periods in a woman's life is the most bone mass accumulated? A) Ages 15-25 B) Ages 25-35 C) Ages 35-45 D) Ages 45-55

A. The time of greatest accumulation of bone mass in women is during adolescence.

A 77-year-old man is brought to your office by his wife, who states that he has been having mental difficulties in recent months, such as not being able to balance their checkbook or plan for his annual visit with the accountant. She also tells you that he has reported seeing animals in the room with him that he can describe vividly. He takes frequent naps and stares blankly for long periods of time. He seems almost normal at times, but randomly appears very confused at other times. He has also been dreaming a lot and has fallen down more than once recently. He currently takes aspirin, 81 mg/d. On examination, the patient walks slowly with a stooped posture and almost falls when turning around. He has only minimal facial expressiveness. No tremor is noted and the remainder of the examination is normal. He is able to recall three words out of three, but clock drawing is abnormal. Laboratory studies are normal and a CT of the brain shows changes of aging. What type of dementia does this patient most likely have? A) Dementia with Lewy bodies B) Alzheimer disease C) Frontotemporal dementia D) Vascular dementia E) Dementia of Parkinson disease

A. This patient has dementia with Lewy bodies, which is the third most common type after Alzheimer disease and vascular dementia. He demonstrates typical signs and symptoms, including well-formed hallucinations, vivid dreams, fluctuating cognition, sleep disorder with periods of daytime sleeping, frequent falls, deficits in visuospatial ability (abnormal clock drawing), and rapid eye movement (REM) sleep disorder (vivid dreams). In Alzheimer disease, the predominant early symptom is memory impairment without the other symptoms found in this patient. In dementia of Parkinson disease, extrapyramidal symptoms such as tremor, bradykinesia, and rigidity precede the onset of memory impairment by more than 1 year. Frontotemporal dementia presents with behavioral changes, including disinhibition, or language problems such as aphasia.

A 75-year-old man presents to the emergency room with the sudden onset of nausea and vomiting. His medical history is notable for coronary artery disease and well-controlled hypertension. On examination he refuses to open his eyes or move his head, but when finally coaxed to sit up, he immediately starts to retch and vomit. Rotational nystagmus is noted. He cannot walk because of the dizziness and nausea that walking evokes. His noncontrast brain computed tomography (CT) scan is read as normal for age. Which of the following is the best next step? A) MRI/magnetic resonance angiography (MRA). B) Obtain a thorough psychosocial history. C) Dix-Hallpike maneuver. D) Prescribe meclizine. E) Referral to neurology.

A. This patient has symptoms of central vertigo. The onset of symptoms was abrupt and severe. His gait is affected. If he were able to cooperate with an examination of his cerebellar functions, it would most likely be abnormal. His age and history of hypertension and coronary artery disease place him at elevated risk for cerebellar infarction or hemorrhage. CT is not the appropriate test for examining the brainstem; MRI is much more accurate. MRA may be useful for delineating the exact vascular cause of the symptoms.

A 45-year-old man was brought to the emergency room (ER) after vomiting bright red blood. He has a blood pressure of 88/46 mm Hg and heart rate of 120 bpm. Which of the following is the best next step? A) Intravenous fluid resuscitation and preparation for a transfusion B) Administration of a proton pump inhibitor C) Guaiac test of the stool D) Treatment for H pylori

A. This patient is hemodynamically unstable with hypotension and tachycardia as a consequence of the acute blood loss. Volume resuscitation, immediately with crystalloid or colloid solution, followed by blood transfusion, if necessary, is the initial step to prevent irreversible shock and death. Later, after stabilization, acid suppression and H pylori treatment might be useful to heal an ulcer, if one is present.

A 49-year-old man is admitted to the intensive care unit (ICU) with a diagnosis of an inferior myocardial infarction. His heart rate is 35 bpm and blood pressure 90/50 mm Hg. His ECG shows a Mobitz type I heart block. Which of the following is the best next step? A) Atropine B) Transvenous pacer C) Lidocaine D) Observation

A. This patient's bradycardia is severe, probably a result of the inferior myocardial infarction. Atropine is the agent of choice in this situation. Mobitz type I block has a good prognosis (vs complete heart block), so transvenous pacing is not usually required.

A 25-year-old medical student is stuck with a hollow needle during a procedure performed on a patient known to have hepatitis B and C viral infection, but who is HIV negative. The student's baseline laboratory studies include serology: HBsAg negative, anti-HBsAb positive, anti-HBc IgG negative. Which of the following regarding this medical student's hepatitis status is true? A) Prior vaccination with hepatitis B vaccine B) Acute infection with hepatitis B virus C) Prior infection with hepatitis B virus D) The student was vaccinated for hepatitis B but is not immune

A. This student's serology is most consistent with vaccination and not prior infection. Like all health care workers, the student should have been vaccinated against the hepatitis B virus, which induces anti-HBs IgG antibody, which is thought to be protective. Not all people receiving the vaccine develop an adequate antibody titer; if none were detected, it would indicate the need for revaccination. Patients with prior hepatitis B infection will also likely have anti-HBsAb but will also have anti-HBc IgG. Acute infection would be signified by the presence of either HBsAg or anti-HBc IgM.

Which one of the following recommendations is accurate regarding the current USPSTF recommendation for osteoporosis screening in the elderly? A) All women with strong risk factors, regardless of age, should be screened for osteoporosis. B) Only women above the age of 65 should be screened for osteoporosis. C) Men and women above the age of 65 should be screened for osteoporosis. D) African-American race is an independent risk factor for osteoporosis and should warrant screening regardless of other risk factors.

A. USPSTF recommends screening for osteoporosis in women above 65 years and younger than 65 years with risk factors. Therefore, considering age as a risk factor, essentially all women with risk factors must be screened for osteoporosis with bone mineral density test or DEXA scan. The current recommendation applies only to women, as there is insufficient evidence to support screening in men, and the race mostly at risk is Caucasian.

A 67-year-old woman with extensive atherosclerotic cerebrovascular disease complains of dizziness and vertigo. Which of the following arteries is most likely to be affected? A) Vertebrobasilar B) Carotid C) Aorta D) Middle cerebral

A. Vertigo and dizziness can be seen in vertebrobasilar insufficiency. Transient monocular blindness or Amaurosis fugx is associated with internal carotid pathology. Face weakness, dysarthria, and hemiplegia greater in the upper extremity is associated with pathology in the middle cerebral artery.

A 55-year-old man is noted to have congestive heart failure and states that he is comfortable at rest but becomes dyspneic even with walking to the bathroom. On echocardiography, he is noted to have an ejection fraction of 47%. Which of the following is the more accurate description of this patient's condition? A) Diastolic dysfunction B) Systolic dysfunction C) Dilated cardiomyopathy D) Pericardial disease

A. When the ejection fraction exceeds 40%, there is likely diastolic dysfunction, with stiff ventricles. The stiff thickened ventricles do not accept blood very readily. This patient has symptoms with mild exertion that are indicative of functional class III. The worst class is level IV, manifested as symptoms at rest or with minimal exertion. ACE inhibitors are important agents in patients with diastolic dysfunction.

A 32-year-old man presents to your clinic with Kayser-Fleischer rings, dysarthria, and spasticity. Pick the cause from the following that is probably responsible for the patient's presentation. A) Wilson disease B) Hematochromatosis C) Primary biliary cirrhosis D) Sclerosing cholangitis E) Autoimmune hepatitis F) Alcohol-induced hepatitis G) Viral hepatitis

A. Wilson disease is an inherited disorder of copper metabolism. The inability to excrete excess copper leads to deposition of the mineral in the liver, brain, and other organs. Patients can present with fulminant hepatitis, acute nonfulminant hepatitis, or cirrhosis, or with bizarre behavioral changes as a result of neurologic damage. Kayser-Fleischer rings develop when copper is released from the liver and deposits in Descemet membrane of the cornea. Patients will often have high copper levels in their urine and low serum ceruloplasmin. Treatment is chelation with penicillamine and pyridoxine supplementation.


Ensembles d'études connexes

Surface Area and Volume: Three dimensional Figures

View Set

Principles of Finance Unit 3- Milestone 3

View Set

ast 109 saturn and debris textbook quizzes

View Set

Mass Comm: Recording/Radio, Television, Online Media

View Set